Chapter 52: Assessment and Management of Patients With Endocrine Disorders

Lakukan tugas rumah & ujian kamu dengan baik sekarang menggunakan Quizwiz!

The nurse is planning the care of a client with hyperthyroidism. What should the nurse specify in the client's meal plan?

Small, frequent meals, high in protein and calories

A nurse is assessing a client with hyperthyroidism. What findings should the nurse expect?

Weight loss, nervousness, and tachycardia

A patient has been diagnosed with Cushing's syndrome. The nurse would expect which of the following features to be present upon physical examination?

"Buffalo hump" Thin extremities "Moon face" Truncal obesity Purple striae

Which hormones are secreted by the posterior lobe of the pituitary gland? Select all that apply. A. Oxytocin B. Vasopressin C. Thyroid-stimulating hormone (TSH) D. Luteinizing hormone (LH) E. Follicle-stimulating hormone (FSH)

A. Oxytocin B. Vasopressin Important hormones secreted by the posterior lobe of the pituitary gland include vasopressin and oxytocin. TSH, FSH, and LH are secreted by the anterior lobe of the pituitary gland.

A young client has a significant height deficit and is to be evaluated for diagnostic purposes. What could be the cause of this client's disorder? A. pituitary disorder B. parathyroid disorder C. thyroid disorder D. adrenal disorder

A. pituitary disorder Pituitary disorders usually result from excessive or deficient production and secretion of a specific hormone. Dwarfism occurs when secretion of growth hormone is insufficient during childhood.

A client with a diagnosis of syndrome of inappropriate antidiuretic hormone secretion (SIADH) is being cared for on the critical care unit. The priority nursing diagnosis for a client with this condition is what?

Excess fluid volume

Which is a clinical manifestation of diabetes insipidus?

Excessive thirst

Patients with hyperthyroidism are characteristically:

Sensitive to heat

The nurse is completing discharge teaching with a client with hyperthyroidism who has been treated with radioactive iodine at an outpatient clinic. The nurse instructs the client to

monitor for symptoms of hypothyroidism.

During a follow-up visit to the physician, a client with hyperparathyroidism asks the nurse to explain the physiology of the parathyroid glands. The nurse states that these glands produce parathyroid

phosphorus.

A nurse cares for an acutely ill client. The nurse understands that the most accurate indicator of fluid loss or gain in an acutely ill client is:

weight.

During a client education session, the nurse describes the role of endocrine glands. Which homeostatic process is regulated by hormones? A. All options are correct. B. growth C. fluid balance D. sleep

A. All options are correct. Hormones circulate in the blood until they reach receptors in target cells or other endocrine glands. They play a vital role in regulating homeostatic processes such as metabolism, growth, fluid and electrolyte balance, reproductive processes, sleep and wake cycles, etc.

Which symptom of thyroid disease is seen in older adults? A. Atrial fibrillation B. Restlessness C. Weight gain D. Hyperactivity

A. Atrial fibrillation Symptoms seen in older adults include weight loss and atrial fibrillation. Older adults may not experience restlessness or hyperactivity.

A nurse is assigned to care for a patient with increased parathormone secretion. Which of the following serum levels should the nurse monitor for this patient? A. Calcium B. Glucose C. Potassium D. Sodium

A. Calcium Increased secretion of parathormone results in bone resorption. Calcium is released into the blood, increasing serum levels.

A nurse is assigned to care for a patient with increased parathormone secretion. Which of the following serum levels should the nurse monitor for this patient? A. Calcium B. Potassium C. Sodium D. Glucose

A. Calcium Increased secretion of parathormone results in bone resorption. Calcium is released into the blood, increasing serum levels.

A client diagnosed with pyelonephritis and possible septicemia has had five urinary tract infections over the past 2 years. He's fatigued from lack of sleep; urinates frequently, even during the night; and has lost weight recently. Tests reveal the following: sodium level 152 mEq/L, osmolarity 340 mOsm/L, glucose level 125 mg/dl, and potassium level 3.8 mEq/L. Which nursing diagnosis is most appropriate for this client? A. Deficient fluid volume related to inability to conserve water B. Imbalanced nutrition: Less than body requirements related to hypermetabolic state C. Deficient fluid volume related to osmotic diuresis induced by hypernatremia D. Imbalanced nutrition: Less than body requirements related to catabolic effects of insulin deficiency

A. Deficient fluid volume related to inability to conserve water The client has signs and symptoms of diabetes insipidus, probably caused by the failure of his renal tubules to respond to antidiuretic hormone as a consequence of pyelonephritis. Therefore, Deficient fluid volume related to inability to conserve water is the most appropriate nursing diagnosis. The client's hypernatremia is secondary to his water loss. Imbalanced nutrition: Less than body requirements — whether it's related to hypermetabolic state or catabolic effect of insulin deficiency — is an inappropriate nursing diagnosis for this client.

Which of the following medications is used in the treatment of diabetes insipidus to control fluid balance? A. Desmopressin (DDAVP) B. Thiazide diuretics C. Diabinese D. Ibuprofen

A. Desmopressin (DDAVP) DDAVP is a synthetic vasopressin used to control fluid balance and prevent dehydration. Other medications that are used in the treatment of patients with diabetes insipidus include Diabinese, thiazide diuretics (potentiate action of vasopressin), and/or prostaglandin inhibitors such as ibuprofen and aspirin.

A client sustained a head injury when falling off of a ladder. While in the hospital, the client begins voiding large amounts of clear urine and states he is very thirsty. The client states that he feels weak, and he has had an 8-lb weight loss since admission. What should the client be tested for? A. Diabetes insipidus (DI) B. Hypothyroidism C. Pituitary tumor D. Syndrome of inappropriate antidiuretic hormone secretion (SIADH)

A. Diabetes insipidus (DI) Urine output may be as high as 20 L/24 hours. Urine is dilute, with a specific gravity of 1.002 or less. Limiting fluid intake does not control urine exertion. Thirst is excessive and constant. Activities are limited by the frequent need to drink and void. Weakness, dehydration, and weight loss develop. SIADH will have the opposite clinical manifestations. The client's symptoms are related to the trauma and not a pituitary tumor. The thyroid gland does not exhibit these symptoms.

A client is being seen in the clinic to receive the results of the lab work to determine thyroid levels. The nurse observes the client's eyes appear to be bulging, and there is swelling around the eyes. What does the nurse know that the correct documentation of this finding is? A. Exophthalmos B. Periorbital swelling C. Bulging eyes D. Retinal detachment

A. Exophthalmos Exophthalmos is an abnormal bulging or protrusion of the eyes and periorbital swelling. These findings are not consistent with retinal detachment.

A nurse is caring for a client with suspected diabetes insipidus. Which test does the nurse anticipate the physician will order to confirm the diagnosis? A. Fluid deprivation test B. Urine glucose test C. Serum ketone test D. Capillary blood glucose test

A. Fluid deprivation test The fluid deprivation test involves withholding water for 4 to 18 hours and periodically checking urine and plasma osmolarity. A client with diabetes insipidus will have an increased serum osmolarity of less than 300 mOsm/kg. Urine osmolarity won't increase. The capillary blood glucose test rapidly measures glucose level in whole blood. The serum ketone test is used to diagnose diabetic ketoacidosis. The urine glucose test monitors glucose levels in urine; however, diabetes insipidus doesn't affect urine glucose levels, so this test isn't appropriate.

A middle-aged female client complains of anxiety, insomnia, weight loss, the inability to concentrate, and eyes feeling "gritty." Thyroid function tests reveal the following: thyroid-stimulating hormone (TSH) 0.02 U/ml, thyroxine 20 g/dl, and triiodothyronine 253 ng/dl. A 6-hour radioactive iodine uptake test showed a diffuse uptake of 85%. Based on these assessment findings, the nurse should suspect: A. Graves' disease. B. multinodular goiter. C. thyroiditis. D. Hashimoto's thyroiditis.

A. Graves' disease. Graves' disease, an autoimmune disease causing hyperthyroidism, is most prevalent in middle-aged females. In Hashimoto's thyroiditis, the most common form of hypothyroidism, TSH levels would be high and thyroid hormone levels low. In thyroiditis, radioactive iodine uptake is low (?2%), and a client with a multinodular goiter will show an uptake in the high-normal range (3% to 10%).

The nurse is aware that the clinical symptoms of a patient with hypoparathyroidism are the result of the initial physiologic response of: A. Hypocalcemia. B. Increased serum levels of phosphate. C. Decreased levels of vitamin D. D. Cardiac arrhythmias.

A. Hypocalcemia. Hypoparathyroidism results in hypocalcemia, which triggers a series of physiologic responses, including the choices presented.

The nurse knows to advise the patient with hyperparathyroidism that he or she should be aware of signs of the common complication of: A. Kidney Stones B. Heart palpitations C. Gastric esophageal reflex D. Bone fractures

A. Kidney Stones The formation of stones in one or both kidneys is caused by the increased urinary excretion of calcium and phosphorus. It occurs in more than 50% of patients with primary hyperparathyroidism. Renal damage causes the kidney stones.

A number of pharmacologic agents are used to treat hyperthyroidism. Which of the following drugs is one of the most commonly prescribed and acts by blocking synthesis of the thyroid hormones? A. Methimazole B. Dexamethasone C. Potassium Iodide D. Propranolol

A. Methimazole Propylthiouracil (PTU) and methimazole are commonly used. They both act by blocking the synthesis of hormones. The other choices suppress the release of the thyroid hormones, except for propranolol which is a beta-adrenergic blocking agent.

A nurse is providing care to a client with primary hyperparathyroidism. Which interventions would be included in the client's care plan? Select all that apply. A. Monitor for fluid overload. B. Encourage intake of dairy products, seafood, nuts, broccoli, and spinach. C. Monitor gait, balance, and fatigue level with ambulation. D. Monitor for signs and symptoms of diarrhea.

A. Monitor for fluid overload. C. Monitor gait, balance, and fatigue level with ambulation. Excessive calcium in the blood depresses the responsiveness of the peripheral nerves, accounting for fatigue and muscle weakness. A large volume of fluid is encouraged to keep the urine dilute. Possible effects include nausea, vomiting, and constipation. Client would be on a calcium-restricted diet.

A nurse is providing care to a client with primary hyperparathyroidism. Which interventions would be included in the client's care plan? Select all that apply. A. Monitor gait, balance, and fatigue level with ambulation. B. Encourage intake of dairy products, seafood, nuts, broccoli, and spinach. C. Monitor for fluid overload. D. Monitor for signs and symptoms of diarrhea.

A. Monitor gait, balance, and fatigue level with ambulation. C. Monitor for fluid overload. Excessive calcium in the blood depresses the responsiveness of the peripheral nerves, accounting for fatigue and muscle weakness. A large volume of fluid is encouraged to keep the urine dilute. Possible effects include nausea, vomiting, and constipation. Client would be on a calcium-restricted diet.

A patient with a history of hypothyroidism is admitted to the intensive care unit unconscious and with a temperature of 95.2ºF. A family member informs the nurse that the patient has not taken thyroid medication in over 2 months. What does the nurse suspect that these findings indicate? A. Myxedema coma B. Diabetes insipidus C. Thyroid storm D. Syndrome of inappropriate antidiuretic hormone (SIADH)

A. Myxedema coma Myxedema coma is a rare life-threatening condition. It is the decompensated state of severe hypothyroidism in which the patient is hypothermic and unconscious (Ross, 2012a). This condition may develop with undiagnosed hypothyroidism and may be precipitated by infection or other systemic disease or by use of sedatives or opioid analgesic agents. Patients may also experience myxedema coma if they forget to take their thyroid replacement medication.

A client visits the clinic to seek treatment for disturbed sleep cycles and depressed mood. Which glands and hormones help to regulate sleep cycles and mood? A. Pineal gland, melatonin B. Parathyroid glands, parathormone C. Adrenal cortex, corticosteroids D. Thymus gland, thymosin

A. Pineal gland, melatonin The pineal gland secretes melatonin, which aids in regulating sleep cycles and mood. Melatonin plays a vital role in hypothalamicpituitary interaction. The thymus gland secretes thymosin and thymopoietin, which aid in developing T lymphocytes. The parathyroid glands secrete parathormone, which increases the levels of calcium and phosphorus in the blood. The adrenal cortex secretes corticosteroids hormones, which influence many organs and structures of the body.

During an assessment of a patient with SIADH, the nurse notes the unexpected result of: A. Pitting edema in the lower extremities. B. A blood pressure reading of 120/85 mm Hg. C. Normal skin turgor. D. Moist mucous membranes.

A. Pitting edema in the lower extremities. In SIADH, the patient does not appear to retain fluids because reabsorbed water is intracellular rather than interstitial.

ADH is secreted by which gland? A. Posterior pituitary B. Thyroid C. Adrenal D. Anterior pituitary

A. Posterior pituitary Antidiuretic hormone (vasopressin) is secreted by the posterior pituitary gland. The anterior pituitary secretes growth hormone. The adrenal gland secretes glucocorticoids and mineralocorticoids. The thyroid gland secretes T3 and T4.

A client with acromegaly is admitted to the hospital with complaints of partial blindness that began suddenly. What does the nurse suspect is occurring with this client? A. Pressure on the optic nerve B. Corneal abrasions C. Glaucoma D. Retinal detachment

A. Pressure on the optic nerve Partial blindness may result from pressure on the optic nerve. Glaucoma does not occur suddenly, and the client did not report injury to suspect corneal abrasions or retinal detachment.

A client is admitted to the health care facility for evaluation for Addison's disease. Which laboratory test result best supports a diagnosis of Addison's disease? A. Serum potassium level of 5.8 mEq/L B. Blood glucose level of 90 mg/dl C. Blood urea nitrogen (BUN) level of 12 mg/dl D. Serum sodium level of 134 mEq/L

A. Serum potassium level of 5.8 mEq/L Addison's disease decreases the production of aldosterone, cortisol, and androgen, causing urinary sodium and fluid losses, an increased serum potassium level, and hypoglycemia. Therefore, an elevated serum potassium level of 5.8 mEq/L best supports a diagnosis of Addison's disease. A BUN level of 12 mg/dl and a blood glucose level of 90 mg/dl are within normal limits. In a client with Addison's disease, the serum sodium level would be much lower than 134 mEq/L, a nearly normal level.

Parathyroid hormone (PTH) has which effects on the kidney? A. Stimulation of calcium reabsorption and phosphate excretion B. Stimulation of calcium reabsorption and phosphate excretion C. Increased absorption of vitamin D and excretion of vitamin E D. Increased absorption of vitamin E and excretion of vitamin D

A. Stimulation of calcium reabsorption and phosphate excretion PTH stimulates the kidneys to reabsorb calcium and excrete phosphate and converts vitamin D to its active form, 1,25-dihydroxyvitamin D. PTH doesn't have a role in the metabolism of vitamin E.

A client is scheduled for a diagnostic test to measure blood hormone levels. The nurse expects that this test will determine which of the following? A. The functioning of endocrine glands B. The concentration of a substance in plasma C. The client's blood sugar level D. Details about the size of the organ and its location

A. The functioning of endocrine glands Measuring blood hormone levels helps determine the functioning of endocrine glands. A radioimmunoassay determines the concentration of a substance in plasma. The measurement of blood hormone levels will not reveal a client's blood sugar level. Radiographs of the chest or abdomen determine the size of the organ and its location.

The nurse auscultates a bruit over the thyroid glands. What does the nurse understand is the significance of this finding? A. The patient may have hyperthyroidism. B. The patient may have thyroiditis. C. The patient may have hypothyroidism. D. The patient may have Cushing disease.

A. The patient may have hyperthyroidism. If palpation discloses an enlarged thyroid gland, both lobes are auscultated using the diaphragm of the stethoscope. Auscultation identifies the localized audible vibration of a bruit. This is indicative of increased blood flow through the thyroid gland associated with hyperthyroidism and necessitates referral to a physician.

A client with thyroiditis has undergone surgery and is concerned about the barely visible scar. Which suggestion should the nurse give the client to cope with the condition? A. Wear clothing that covers the neck B. Undergo a skin graft C. Apply medicines to remove the scar D. Consider cosmetic surgery

A. Wear clothing that covers the neck The nurse may suggest that the client wear clothing that covers the neck and assure the client that the scar is almost invisible. Application of medicines, skin graft, and cosmetic surgery are not appropriate suggestions.

A nurse is assessing a client with hyperthyroidism. What findings should the nurse expect? A. Weight loss, nervousness, and tachycardia B. Weight gain, constipation, and lethargy C. Diaphoresis, fever, and decreased sweating D. Exophthalmos, diarrhea, and cold intolerance

A. Weight loss, nervousness, and tachycardia Weight loss, nervousness, and tachycardia are signs of hyperthyroidism. Other signs of hyperthyroidism include exophthalmos, diaphoresis, fever, and diarrhea. Weight gain, constipation, lethargy, decreased sweating, and cold intolerance are signs of hypothyroidism.

What clinical manifestations does the nurse recognize would be associated with a diagnosis of hyperthyroidism? Select all that apply. A. Weight loss. B. A pulse rate slower than 90 bpm C. Intolerance to cold D. An elevated systolic blood pressure E. Muscular fatigability

A. Weight loss. D. An elevated systolic blood pressure E. Muscular fatigability Manifestations of hyperthyroidism include an increased appetite and dietary intake, weight loss, fatigability and weakness (difficulty in climbing stairs and rising from a chair), amenorrhea, and changes in bowel function. Atrial fibrillation occurs in 15% of in older adult patients with new-onset hyperthyroidism (Porth & Matfin, 2009). Cardiac effects may include sinus tachycardia or dysrhythmias, increased pulse pressure, and palpitations. These patients are often emotionally hyperexcitable, irritable, and apprehensive; they cannot sit quietly; they suffer from palpitations; and their pulse is abnormally rapid at rest as well as on exertion. They tolerate heat poorly and perspire unusually freely.

For the first 72 hours after thyroidectomy surgery, a nurse should assess a client for Chvostek's sign and Trousseau's sign because they indicate: A. hypocalcemia B. hypokalemia. C. hypercalcemia. D. hyperkalemia.

A. hypocalcemia A client who has undergone a thyroidectomy is at risk for developing hypocalcemia from inadvertent removal of or damage to the parathyroid gland. The client with hypocalcemia will exhibit a positive Chvostek's sign (facial muscle contraction when the facial nerve in front of the ear is tapped) and a positive Trousseau's sign (carpal spasm when a blood pressure cuff is inflated for a few minutes). These signs aren't present with hypercalcemia, hypokalemia, or hyperkalemia.

A client is undergoing diagnostics for an alteration in thyroid function. What physiologic function is affected by altered thyroid function? A. metabolic rate B. fluid/electrolyte balance C. growth D. sleep/wake cycles

A. metabolic rate The thyroid concentrates iodine from food and uses it to synthesize thyroxine (T4) and triiodothyronine (T3). These two hormones regulate the body's metabolic rate.

An incoherent client with a history of hypothyroidism is brought to the emergency department by the rescue squad. Physical and laboratory findings reveal hypothermia, hypoventilation, respiratory acidosis, bradycardia, hypotension, and nonpitting edema of the face and periorbital area. Knowing that these findings suggest severe hypothyroidism, the nurse prepares to take emergency action to prevent the potential complication of: A. myxedema coma. B. cretinism. C. thyroid storm. D. Hashimoto's thyroiditis.

A. myxedema coma. Severe hypothyroidism may result in myxedema coma, in which a drastic drop in the metabolic rate causes decreased vital signs, hypoventilation (possibly leading to respiratory acidosis), and nonpitting edema. Thyroid storm is an acute complication of hyperthyroidism. Cretinism is a form of hypothyroidism that occurs in infants. Hashimoto's thyroiditis is a common chronic inflammatory disease of the thyroid gland in which autoimmune factors play a prominent role.

A client with a history of pheochromocytoma is admitted to the hospital in an acute hypertensive crisis. To reverse hypertensive crisis caused by pheochromocytoma, the nurse expects to administer: A. phentolamine (Regitine). B. felodipine (Plendil). C. methyldopa (Aldomet). D. mannitol (Osmitrol).

A. phentolamine (Regitine). Pheochromocytoma causes excessive production of epinephrine and norepinephrine, natural catecholamines that raise the blood pressure. Phentolamine, an alpha-adrenergic given by I.V. bolus or drip, antagonizes the body's response to circulating epinephrine and norepinephrine, reducing blood pressure quickly and effectively. Although methyldopa is an antihypertensive agent available in parenteral form, it isn't effective in treating hypertensive emergencies. Mannitol, a diuretic, isn't used to treat hypertensive emergencies. Felodipine, an antihypertensive agent, is available only in extended-release tablets and therefore doesn't reduce blood pressure quickly enough to correct hypertensive crisis.

A nurse is caring for a client with a kidney disorder. What hormone released by the kidneys initiates the production of angiotensin and aldosterone to increase blood pressure and blood volume? A. renin B. gastrin C. cholecystokinin D. erythropoietin

A. renin Renin is released from the kidneys and initiates the production of angiotensin and aldosterone to increase blood pressure and blood volume. The kidneys secrete erythropoietin, a substance that promotes the maturation of red blood cells. Cholecystokinin released from cells in the small intestine stimulates contraction of the gallbladder to release bile when dietary fat is ingested. Gastrin is released within the stomach to increase the production of hydrochloric acid.

A group of students is reviewing material about endocrine system function. The students demonstrate understanding of the information when they identify which of the following as secreted by the adrenal medulla? A. Glucocorticoids B. Epinephrine C. Mineralocorticoids D. Glucagon

B. Epinephrine The adrenal medulla secretes epinephrine and norepinephrine. The adrenal cortex manufactures and secretes glucocorticoids, mineralocorticoids, and small amounts of androgenic sex hormones. Glucagon is released by the pancreas.

Which of the following would the nurse expect to find in a client with severe hyperthyroidism? A. Buffalo hump B. Exophthalmos C. Striae D. Tetany

B. Exophthalmos Exophthalmos that results from enlarged muscle and fatty tissue surrounding the rear and sides of the eyeball is seen in clients with severe hyperthyroidism. Tetany is the symptom of acute and sudden hypoparathyroidism. Buffalo hump and striae are the symptoms of Cushing's syndrome.

Which of the following precautions would be most appropriate when caring for a client being treated with radioactive iodine (RAI) for a thyroid tumor? A. Monitor the respiratory status. B. Handle body fluids carefully. C. Administer prescribed corticosteroids carefully. D. Administer the prescribed medications at the same time each day.

B. Handle body fluids carefully. The nurse handles body fluids carefully to prevent spread of contamination. Corticosteroids are not prescribed for thyroid tumor. Monitoring the respiratory status and administering prescribed medicines at the same time each day are unrelated to the care of a client receiving RAI.

A client has been diagnosed with nephrogenic diabetes insipidus (DI), and the physician is initiating treatment. What medication does the nurse prepare to administer for this client? A. Furosemide B. Hydrochlorothiazide C. Bumetanide D. Metolazone

B. Hydrochlorothiazide The physician prescribes a thiazide diuretic, such as hydrochlorothiazide. The thiazide acts at the proximal convoluted tubule, leaving less fluid for excretion in the distal convoluted tubules, the portion affected by nephrogenic diabetes insipidus (DI). Consequently, the client excretes water, but the total volume is less than in an untreated state. The other diuretics listed do not work on the proximal convoluted tubule and would not be effective in treatment.

An older adult female client has been complaining of sleeping more, increased urination, anorexia, weakness, irritability, depression, and bone pain that interferes with her going outdoors. Based on these assessment findings, the nurse should suspect which disorder? A. Hypoparathyroidism B. Hyperparathyroidism C. Diabetes insipidus D. Diabetes mellitus

B. Hyperparathyroidism Hyperparathyroidism is most common in older women and is characterized by bone pain and weakness from excess parathyroid hormone. Clients also exhibit hypercalciuria-causing polyuria. Although clients with diabetes mellitus and diabetes insipidus have polyuria, they don't have bone pain and increased sleeping. Hypoparathyroidism is characterized by urinary frequency rather than by polyuria.

A patient has been placed on corticosteroid therapy for an Addison's disease. The nurse should be aware of which of the following side effects with this type of therapy? Select all that apply. A. Hypotension B. Hypertension C. Poor wound healing D. Weight loss E. Alterations in glucose metabolism

B. Hypertension C. Poor wound healing E. Alterations in glucose metabolism Side effects of corticosteroid therapy include hypertension, alterations in glucose metabolism, weight gain, and poor wound healing.

Which condition may occur during the postoperative period in a client who underwent adrenalectomy because of sudden withdraw of excessive amounts of catecholamines? A. Hyperglycemia B. Hypoglycemia C. Hyporeflexia D. Hypertension

B. Hypoglycemia Hypotension and hypoglycemia may occur in the postoperative period because of the sudden withdrawal of excessive amounts of catecholamines. Hypertension and hyporeflexia are not related to the sudden withdraw of excessive amounts of catecholamines.

A client is admitted to the hospital and will be undergoing tests to determine if he has an abdominal mass. What should the nurse be sure to document when asking about allergies? A. If the client is allergic to beef B. If the client is allergic to seafood C. If the client is allergic to grapefruit D. If the client is allergic to pork

B. If the client is allergic to seafood The nurse documents an allergy to iodine, a component of contrast dyes, or seafood, and informs the physician. Pork, beef, and grapefruit do not interact with the contrast dye that the client will receive during testing.

The nurse knows to advise the patient with hyperparathyroidism that he or she should be aware of signs of the common complication of: A. Gastric esophageal reflex B. Kidney Stones C. Heart palpitations D. Bone fractures

B. Kidney Stones The formation of stones in one or both kidneys is caused by the increased urinary excretion of calcium and phosphorus. It occurs in more than 50% of patients with primary hyperparathyroidism. Renal damage causes the kidney stones.

After teaching a class about the endocrine system, the instructor determines that the students need additional instruction when they identify which of the following as an endocrine gland? A.Testes B. Kidneys C. Adrenal gland D. Pancreas

B. Kidneys Although the kidneys secrete renin and erythropoietin, they are typically not considered endocrine glands. The pancreas, adrenal glands, and testes are considered endocrine glands.

Which of the following would the nurse expect the physician to order for a client with hypothyroidism? A. Propranolol B. Levothyroxine sodium C. Propylthiouracil D. Methimazole

B. Levothyroxine sodium Hypothyroidism is treated with thyroid replacement therapy, in the form of dessicated thyroid extract or a synthetic product, such as levothyroxine sodium (Synthroid) or liothyronine sodium (Cytomel). Methimazole and propylthiouracil are antithyroid agents used to treat hyperthyroidism. Propranolol is a beta blocker that can be used to treat hyperthyroidism.

A client has a dysfunction in one of his glands that is causing a decrease in the level of calcium in the blood. What gland should be evaluated for dysfunction? A. Thyroid gland B. Parathyroid gland C. Thymus gland D. Adrenal gland

B. Parathyroid gland The parathyroids secrete parathormone, which increases the level of calcium in the blood when there is a decrease in the serum level. The thyroid, thymus, and adrenal gland do not secrete calcium.

The nurse is caring for a client with hypoparathyroidism. When the nurse taps the client's facial nerve, the client's mouth twitches and the jaw tightens. What is this response documented as related to the low calcium levels? A. Positive Trousseau's sign B. Positive Chvostek's sign C. Positive Babinski's sign D. Positive paresthesias

B. Positive Chvostek's sign If a nurse taps the client's facial nerve (which lies under the tissue in front of the ear), the client's mouth twitches and the jaw tightens. The response is identified as a positive Chvostek's sign. A positive Trousseau's sign is elicited by placing a BP cuff on the upper arm, inflating it between the systolic and diastolic BP, and waiting 3 minutes. The nurse observes the client for spasm of the hand (carpopedal spasm), which is evidenced by the hand flexing inward. Positive Babinski's sign is elicited by stroking the sole of the foot. Paresthesia is not a symptom that can be elicited; it is felt by the client.

The nurse is administering a medication to a client with hyperthyroidism to block the production of thyroid hormone. The client is not a candidate for surgical intervention at this time. What medication should the nurse administer to the client? A. Spironolactone B. Propylthiouracil C. Propranolol D. Levothyroxine

B. Propylthiouracil Antithyroid drugs, such as propylthiouracil and methimazole are given to block the production of thyroid hormone preoperatively or for long-term treatment for clients who are not candidates for surgery or radiation treatment. Levothyroxine would increase the level of thyroid and be contraindicated in this client. Spironolactone is a diuretic and does not have the action of blocking production of thyroid hormone and neither does propranolol, which is a beta-blocker.

A client with a history of chronic hyperparathyroidism admits to being noncompliant. Based on initial assessment findings, the nurse formulates the nursing diagnosis of Risk for injury.To complete the nursing diagnosis statement for this client, which "related-to" phrase should the nurse add? A. Related to exhaustion secondary to an accelerated metabolic rate B. Related to bone demineralization resulting in pathologic fractures C. Related to tetany secondary to a decreased serum calcium level D. Related to edema and dry skin secondary to fluid infiltration into the interstitial spaces

B. Related to bone demineralization resulting in pathologic fractures Poorly controlled hyperparathyroidism may cause an elevated serum calcium level. This increase, in turn, may diminish calcium stores in the bone, causing bone demineralization and setting the stage for pathologic fractures and a risk for injury. Hyperparathyroidism doesn't accelerate the metabolic rate. A decreased thyroid hormone level, not an increased parathyroid hormone level, may cause edema and dry skin secondary to fluid infiltration into the interstitial spaces. Hyperparathyroidism causes hypercalcemia, not hypocalcemia; therefore, it isn't associated with tetany.

Which of the following endocrine disorder causes the patient to have dilutional hyponatremia? A. Diabetes insipidus (DI) B. Syndrome of inappropriate antidiuretic hormone secretion (SIADH) C. Hypothyroidism D. Hyperthyroidism

B. Syndrome of inappropriate antidiuretic hormone secretion (SIADH) Patients diagnosed with SIADH retain water and develop a subsequent sodium deficiency known as dilutional hyponatremia. In DI, there is excessive thirst and large volumes of dilute urine. Patients with DI, hypothyroidism, or hyperthyroidism do not exhibit dilutional hyponatremia

The nurse is attempting to locate the thyroid gland in order to determine if it is enlarged. Where should the nurse palpate the thyroid gland? A. The upper neck posterior to the trachea B. The lower neck anterior to the trachea C. Mid trachea D. Distal to the carotid arteries

B. The lower neck anterior to the trachea The thyroid gland is located in the lower neck anterior to the trachea. It is divided into two lateral lobes joined by a band of tissue called the isthmus.

Early this morning, a client had a subtotal thyroidectomy. During evening rounds, the nurse assesses the client (who now has nausea) and records a temperature of 105°F (40.5°C), tachycardia, and extreme restlessness. What is the most likely cause of these signs? A. Tetany B. Thyroid crisis C. Diabetic ketoacidosis D. Hypoglycemia

B. Thyroid crisis Thyroid crisis usually occurs in the first 12 hours after thyroidectomy and causes exaggerated signs of hyperthyroidism, such as high fever, tachycardia, and extreme restlessness. Diabetic ketoacidosis is more likely to produce polyuria, polydipsia, and polyphagia. Hypoglycemia is likely to produce weakness, tremors, profuse perspiration, and hunger. Tetany typically causes uncontrollable muscle spasms, stridor, cyanosis, and possibly asphyxia.

Which hormone is secreted by the posterior pituitary? A. Calcitonin B. Vasopressin C. Somatostatin D. Corticosteroids

B. Vasopressin Vasopressin causes smooth muscle, particularly blood vessels, to contract. Calcitonin is secreted by the parafollicular cells of the thyroid gland. Corticosteroids are secreted by the adrenal cortex. Somatostatin is released by the anterior lobe of the pituitary.

A nurse is assessing a client with hyperthyroidism. What findings should the nurse expect? A. Exophthalmos, diarrhea, and cold intolerance B. Weight loss, nervousness, and tachycardia C. Weight gain, constipation, and lethargy D. Diaphoresis, fever, and decreased sweating

B. Weight loss, nervousness, and tachycardia Weight loss, nervousness, and tachycardia are signs of hyperthyroidism. Other signs of hyperthyroidism include exophthalmos, diaphoresis, fever, and diarrhea. Weight gain, constipation, lethargy, decreased sweating, and cold intolerance are signs of hypothyroidism.

The physician has ordered an outpatient dexamethasone suppression test to diagnose the cause of Cushing syndrome in a client who works at night, from 11:00 PM to 7:00 AM, and normally sleeps from 8:00 AM to 4:00 PM. The client has been given the dexamethasone. To ensure the most reliable test results, the nurse arranges for the plasma cortisol concentration to be tested at which time? A. 12:00 PM B. 8:00 PM C. 5:00 PM D. 8:00 AM

C. 5:00 PM An overnight dexamethasone suppression test is used to diagnose pituitary and adrenal causes of Cushing syndrome. It can be performed on an outpatient basis. Dexamethasone is administered orally late in the evening or at bedtime, and a plasma cortisol concentration is measured at 8 AM the next day. However, in a client who sleeps during the day, the medication would be given before bed and the plasma concentration would be measured soon after awakening in the late afternoon.

The nurse is reviewing a client's laboratory studies and determines that the client has an elevated calcium level. What does the nurse know will occur as a result of the rise in the serum calcium level? A. A rise in serum calcium stimulates the release of T lymphocytes. B. A rise in serum calcium stimulates the release of erythropoietin. C. A rise in serum calcium stimulates the release of calcitonin from the thyroid gland. D. A rise in serum calcium inhibits the release of calcitonin.

C. A rise in serum calcium stimulates the release of calcitonin from the thyroid gland. Calcitonin, another thyroid hormone, inhibits the release of calcium from bone into the extracellular fluid. A rise in the serum calcium level stimulates the release of calcitonin from the thyroid gland.

Which of the following hormones controls secretion of adrenal androgens? A. Thyroid-stimulating hormone (TSH) B. Calcitonin C. Adrenocorticotropic hormone (ACTH) D. Parathormone

C. Adrenocorticotropic hormone (ACTH) ACTH controls the secretion of adrenal androgens. When secreted in normal amounts, the adrenal androgens appear to have little effect, but when secreted in excess, as in certain inborn enzyme deficiencies, masculinization may result. The secretion of T3 and T4 by the thyroid gland is controlled by TSH. Parathormone regulates calcium and phosphorous metabolism. Calcitonin reduces the plasma level of calcium by increasing its deposition in bone.

A client with severe head trauma sustained in a car accident is admitted to the intensive care unit. Thirty-six hours later, the client's urine output suddenly rises above 200 ml/hour, leading the nurse to suspect diabetes insipidus. Which laboratory findings support the nurse's suspicion of diabetes insipidus? A. Below-normal urine and serum osmolality levels B. Above-normal urine and serum osmolality levels C. Below-normal urine osmolality level, above-normal serum osmolality level D. Above-normal urine osmolality level, below-normal serum osmolality lev

C. Below-normal urine osmolality level, above-normal serum osmolality level In diabetes insipidus, excessive polyuria causes dilute urine, resulting in a below-normal urine osmolality level. At the same time, polyuria depletes the body of water, causing dehydration that leads to an above-normal serum osmolality level. For the same reasons, diabetes insipidus doesn't cause above-normal urine osmolality or below-normal serum osmolality levels.

Surgical removal of the thyroid gland is the treatment of choice for thyroid cancer. During the immediate postoperative period, the nurse knows to evaluate serum levels of __________ to assess for a serious and primary postoperative complication of thyroidectomy. A. Potassium B. Magnesium C. Calcium D. Sodium

C. Calcium Efforts are made to spare parathyroid tissue to reduce the risk of postoperative hypocalcemia with resultant tetany.

After undergoing a thyroidectomy, a client develops hypocalcemia and tetany. Which electrolyte should the nurse anticipate administering? A. Potassium chloride B. Sodium phosphorus C. Calcium gluconate D. Sodium bicarbonate

C. Calcium gluconate Immediate treatment for a client who develops hypocalcemia and tetany after thyroidectomy is calcium gluconate. Potassium chloride and sodium bicarbonate aren't indicated. Sodium phosphorus wouldn't be given because phosphorus levels are already elevated.

Trousseau's sign is elicited by which of the following? A. A sharp tapping over the facial nerve just in front of the parotid gland and anterior to the ear causes spasm or twitching of the mouth, nose, and eye. B. After making a clenched fist, the palm remains blanched when pressure is placed over the radial artery. C. Carpopedal spasm is induced by occluding the blood flow to the arm for 3 minutes with the use of a blood pressure cuff. D. The patient complains of pain in the calf when his foot is dorsiflexed.

C. Carpopedal spasm is induced by occluding the blood flow to the arm for 3 minutes with the use of a blood pressure cuff. A positive Trousseau's sign is suggestive of latent tetany. A positive Chvostek's sign is demonstrated when a sharp tapping over the facial nerve just in front of the parotid gland and anterior to the ear causes spasm or twitching of the mouth, nose, and eye. A positive Allen's test is demonstrated by the palm remaining blanched with the radial artery occluded. The radial artery should not be used for an arterial puncture. A positive Homans' sign is demonstrated when the patient complains of pain in the calf when his foot is dorsiflexed.

The primary function of the thyroid gland includes which of the following? A. Reabsorption of water B. Facilitation of milk ejection C. Control of cellular metabolic activity D. Reduction of plasma level of calcium

C. Control of cellular metabolic activity The primary function of the thyroid hormone is to control cellular metabolic activity. Oxytocin facilitates milk ejection during lactation and increases the force of uterine contraction during labor and delivery. Antidiuretic hormone (ADH) release results in reabsorption of water into the bloodstream rather than excretion by the kidneys. Calcitonin reduces the plasma level of calcium by increasing its deposition in bone.

A client with a traumatic brain injury is producing an abnormally large volume of dilute urine. Which alteration to a hormone secreted by the posterior pituitary would the nurse expect to find? A. Increased oxytocin B. A deficient amount of somatostatin C. Deficient production of vasopressin D. Increased antidiuretic hormone

C. Deficient production of vasopressin The most common disorder related to posterior lobe dysfunction is diabetes insipidus, a condition in which abnormally large volumes of dilute urine are excreted as a result of deficient production of vasopressin. Diabetes insipidus may occur following surgical treatment of a brain tumor, secondary to nonsurgical brain tumors, and traumatic brain injury.

The nurse obtains a complete family history of a client with a suspected endocrine disorder based on which rationale? A. Diet and drug histories are related to the family history. B. An allergy to iodine is inherited. C. Endocrine disorders can be inherited. D. It helps determine the client's general status.

C. Endocrine disorders can be inherited. Some endocrine disorders are inherited or have a tendency to run in families. Therefore, it is essential to take a complete family history. A complete blood count and chemistry profile are performed to determine the client's general status and to rule out disorders. Obtaining information about an allergy to iodine is important because diagnostic testing may involve the use of contrast dyes. However, an allergy to iodine is not related to endocrine disorders. Diet and drug histories, although important information, are not associated with the family history.

Which assessment is done by the nurse when conducting a physical examination? A. Determine the client's ability to participate in the assessment B. Examine outstretched hands for skin breaks C. Examine the shape and color of the nails D. Palpate the thyroid gland repeatedly

C. Examine the shape and color of the nails During physical examination, the nurse examines the shape and color of the nails and determines whether they are thin, thick, or brittle. The outstretched hands should be examined for tremors. Repeated palpation of the thyroid gland may have serious implications.

A nurse should perform which intervention for a client with Cushing's syndrome? A. Offer clothing or bedding that's cool and comfortable. B. Explain the rationale for increasing salt and fluid intake in times of illness, increased stress, and very hot weather. C. Explain that the client's physical changes are a result of excessive corticosteroids. D. Suggest a high-carbohydrate, low-protein diet.

C. Explain that the client's physical changes are a result of excessive corticosteroids. The nurse should explain to the client that Cushing's syndrome causes physical changes related to excessive corticosteroids. Clients with hyperthyroidism, not Cushing's syndrome, are heat intolerant and must have cool clothing and bedding. Clients with Cushing's syndrome should have a high-protein, not low-protein, diet. Clients with Addison's disease must increase sodium intake and fluid intake in times of stress of prevent hypotension.

Which outcome indicates that treatment of a client with diabetes insipidus has been effective? A. Blood pressure is 90/50 mm Hg. B. Urine output measures more than 200 ml/hour. C. Fluid intake is less than 2,500 ml/day. D. Heart rate is 126 beats/minute.

C. Fluid intake is less than 2,500 ml/day. diabetes insipidus is characterized by polyuria (up to 8 L/day), constant thirst, and an unusually high oral intake of fluids. Treatment with the appropriate drug should decrease both oral fluid intake and urine output. A urine output of 200 ml/hour indicates continuing polyuria. A blood pressure of 90/50 mm Hg and a heart rate of 126 beats/minute indicate compensation for the continued fluid deficit, suggesting that treatment hasn't been effective.

The nurse is reviewing a client's history which reveals that the client has had an over secretion of growth hormone (GH) that occurred before puberty. The nurse interprets this as which of the following? A. Simmonds' disease B. Acromegaly C. Gigantism D. Dwarfism

C. Gigantism When over secretion of GH occurs before puberty, gigantism results. Dwarfism occurs when secretion of GH is insufficient during childhood. Oversecretion of GH during adulthood results in acromegaly. An absence of pituitary hormonal activity causes Simmonds' disease.

Which hormone would be responsible for increasing blood glucose levels by stimulating glycogenolysis? A. Insulin B. Somatostatin C. Glucagon D. Cholecystokinin

C. Glucagon Glucagon is a hormone released by the alpha islet cells of the pancreas that raises blood glucose levels by stimulating glycogenolysis (the breakdown of glycogen into glucose in the liver). Somatostatin is a hormone secreted by the delta islet cells that helps to maintain a relatively constant level of blood glucose by inhibiting the release of insulin and glucagons. Insulin is a hormone released by the beta islet cells that lowers the level of blood glucose when it rises beyond normal limits. Cholecystokinin is released from the cells of the small intestine that stimulates contraction of the gall bladder to release bile when dietary fat is ingested.

The nurse is caring for a client with diabetes who developed hypoglycemia. What can the nurse administer to the client to raise the blood sugar level? A. Cortisone B. Insulin C. Glucagon D. Estrogen

C. Glucagon Glucagon, a hormone released by alpha islet cells, raises blood sugar levels by stimulating glycogenolysis, the breakdown of glycogen into glucose, in the liver. Insulin is released to lower the blood sugar levels. Cortisone and estrogen are not released from the pancreas.

The adrenal cortex is responsible for producing which substances? A. Norepinephrine and epinephrine B. Mineralocorticoids and catecholamines C. Glucocorticoids and androgens D. Catecholamines and epinephrine

C. Glucocorticoids and androgens The adrenal glands have two divisions, the cortex and medulla. The cortex produces three types of hormones: glucocorticoids, mineralocorticoids, and androgens. The medulla produces catecholamines — epinephrine and norepinephrine.

A woman with a progressively enlarging neck comes into the clinic. She mentions that she has been in a foreign country for the previous 3 months and that she didn't eat much while she was there because she didn't like the food. She also mentions that she becomes dizzy when lifting her arms to do normal household chores or when dressing. What endocrine disorder should the nurse expect the physician to diagnose? A. Diabetes insipidus B. Cushing's syndrome C. Goiter D. Diabetes mellitus

C. Goiter A goiter can result from inadequate dietary intake of iodine associated with changes in foods or malnutrition. It's caused by insufficient thyroid gland production and depletion of glandular iodine. Signs and symptoms of this malfunction include an enlarged thyroid gland, dizziness when raising the arms above the head, dysphagia, and respiratory distress. Signs and symptoms of diabetes mellitus include polydipsia, polyuria, and polyphagia. Signs and symptoms of diabetes insipidus include extreme polyuria (4 to 16 L/day) and symptoms of dehydration (poor tissue turgor, dry mucous membranes, constipation, dizziness, and hypotension). Cushing's syndrome causes buffalo hump, moon face, irritability, emotional lability, and pathologic fractures.

A client with acromegaly has been given the option of a surgical approach or a medical approach. The client decides to have a surgical procedure to remove the pituitary gland. What does the nurse understand this surgical procedure is called? A. Hysteroscopy B. Ablation C. Hypophysectomy D. Thyroidectomy

C. Hypophysectomy The treatment of choice is surgical removal of the pituitary gland (transsphenoidal hypophysectomy) through a nasal approach. The surgeon may substitute an endoscopic technique using microsurgical instruments to reduce surgical trauma. A hysteroscopy is a gynecologic procedure. The thyroid gland is not involved for a surgical procedure. Ablation is not a removal of the pituitary gland.

Which nursing diagnosis takes highest priority for a client with hyperthyroidism? A. Disturbed body image related to weight gain and edema B. Risk for impaired skin integrity related to edema, skin fragility, and poor wound healing C. Imbalanced nutrition: Less than body requirements related to thyroid hormone excess D. Risk for imbalanced nutrition: More than body requirements related to thyroid hormone excess

C. Imbalanced nutrition: Less than body requirements related to thyroid hormone excess In the client with hyperthyroidism, excessive thyroid hormone production leads to hypermetabolism and increased nutrient metabolism. These conditions may result in a negative nitrogen balance, increased protein synthesis and breakdown, decreased glucose tolerance, and fat mobilization and depletion. These changes put the client at risk for marked nutrient and calorie deficiency, making Imbalanced nutrition: Less than body requirements related to thyroid hormone excess the most important nursing diagnosis. Risk for impaired skin integrity related to edema, skin fragility, and poor wound healing and Disturbed body image related to weight gain and edema may be appropriate for a client with hypothyroidism, which slows the metabolic rate.

When reviewing laboratory results for a patient with a possible diagnosis of hypoparathyroidism, the nurse knows that this condition is characterized by which of the following? A. Increase in serum calcium B. Lowered blood phosphate C. Inadequate secretion of parathormone D. Increase in the renal excretion of phosphate

C. Inadequate secretion of parathormone In hypoparathyroidism, there is an increased blood phosphate. Blood calcium is decreased, and there is a decreased renal excretion of phosphate. The secretion of parathormone is inadequate.

A client is receiving long-term treatment with high-dose corticosteroids. Which of the following would the nurse expect the client to exhibit? A. Hypotension B. Pale thick skin C. Moon face D. Weight loss

C. Moon face Clients who are receiving long-term high-dose corticosteroid therapy often develop a cushingoid appearance, manifested by facial fullness and the characteristic moon face. They also may exhibit weight gain, peripheral edema, and hypertension due to sodium and water retention. The skin is usually thin, and ruddy.

Which action would be appropriate when evaluating a client's neck for thyroid enlargement? A. Perform repeated palpation of the thyroid gland. B. Examine the skin of the neck for excessive oiliness. C. Palpate the thyroid gland gently. D. Inspect changes in pigmentation in the neck.

C. Palpate the thyroid gland gently. The nurse should inspect the neck for thyroid enlargement and gently palpate the thyroid gland. Repeated palpation of the thyroid in case of thyroid hyperactivity can result in a sudden release of a large amount of thyroid hormones, which may have serious implications. Pigment changes in the neck and excessive oiliness of the skin are not related to assessment for thyroid enlargement.

Which glands regulate calcium and phosphorous metabolism? A. Pituitary B. Thyroid C. Parathyroid D. Adrenal

C. Parathyroid Parathormone (parathyroid hormone), the protein hormone produced by the parathyroid glands, regulates calcium and phosphorous metabolism. The thyroid gland controls cellular metabolic activity. The adrenal medulla at the center of the adrenal gland secretes catecholamines, and the outer portion of the gland, the adrenal cortex, secretes steroid hormones. The pituitary gland secretes hormones that control the secretion of additional hormones by other endocrine glands.

Vision and visual fields are altered in disorders of which of the following endocrine glands? A. Pancreas B. Thyroid C. Pituitary D. Parathyroid

C. Pituitary The pituitary gland is located close to the optic nerves and hence causes pressure on these nerves; thus, changes in the vision and the visual fields may occur.

What breakfast items would the nurse recommend when assisting with the breakfast menu for a patient with hyperthyroidism? A. Cereal with milk and bananas B. Orange juice and toast C. Pork sausage and cranberry juice D. Fried eggs and bacon

C. Pork sausage and cranberry juice Foods and fluids are selected to replace fluid lost through diarrhea and diaphoresis and to control the diarrhea that results from increased peristalsis. Rapid movement of food through the GI tract may result in nutritional imbalance and further weight loss. To reduce diarrhea, highly seasoned foods and stimulants such as coffee, tea, cola, and alcohol are discouraged. High-calorie, high-protein foods are encouraged.

A patient has been taking tricyclic antidepressants for many years for the treatment of depression. The patient has developed SIADH and has been admitted to the acute care facility. What should the nurse carefully monitor when caring for this patient? Select all that apply. A. Liver function tests B. Signs of dehydration C. Strict intake and output D. Neurologic function E. Urine and blood chemistry

C. Strict intake and output D. Neurologic function E. Urine and blood chemistry Close monitoring of fluid intake and output, daily weight, urine and blood chemistries, and neurologic status is indicated for the patient at risk for SIADH.

Which laboratory test results should a nurse expect to find in a client diagnosed with Hashimoto's thyroiditis? A. T4, 2 µg/dl; T3, 200 ng/dl; TSH 5.9 mIU/ml B. Thyroxine (T4), 22 µg/dl; triiodothyronine (T3), 320 ng/dl; thyroid-stimulating hormone (TSH) undetectable C. T4, 2 µg/dl; T3, 35 ng/dl; TSH 45 mIU/ml D. T4, 22 µg/dl; T3, 200 ng/dl; TSH 0.1 mIU/ml

C. T4, 2 µg/dl; T3, 35 ng/dl; TSH 45 mIU/ml Normal thyroid function tests are as follows: T4, 5 to 12 µg/dl; T3, 65 to 195 ng/dl; TSH 0.3 to 5.4 mIU/ml. With Hashimoto's thyroiditis, T4 and T3 levels are typically subnormal and TSH is elevated. With primary hyperthyroidism, T4 and T3 levels are elevated and TSH is subnormal. With hypothyroidism, T4 is subnormal and T3 and TSH levels are elevated.

A client is returned to his room after a subtotal thyroidectomy. Which piece of equipment is most important for the nurse to keep at the client's bedside? A. Indwelling urinary catheter kit B. Cardiac monitor C. Tracheostomy set D. Humidifier

C. Tracheostomy set After a subtotal thyroidectomy, swelling of the surgical site (the tracheal area) may obstruct the airway. Therefore, the nurse should keep a tracheostomy set at the client's bedside in case of a respiratory emergency. Although an indwelling urinary catheter and a cardiac monitor may be used for a client after a thyroidectomy, the tracheostomy set is more important. A humidifier isn't indicated for this client.

Which diagnostic test is done to determine suspected pituitary tumor? A. radiographs of the abdomen B. radioimmunoassay C. computed tomography scan D. measurement of blood hormone levels

C. computed tomography scan A computed tomography or magnetic resonance imaging scan is done to detect a suspected pituitary tumor. Radiographs of the chest or abdomen are taken to detect tumors. Radiographs also determine the size of the organ and their location. Measuring blood hormone levels helps determine the functioning of endocrine glands. A radioimmunoassay determines the concentration of a substance in plasma.

A nurse explains the role of the ovaries. Which hormones would be included in that discussion? A. testosterone and progesterone B. estrogen and testosterone C. estrogen and progesterone D. estrogen and progestin

C. estrogen and progesterone The ovaries produce estrogen and progesterone. Progestin is a synthetic compound. Testosterone is involved with the development and maintenance of male secondary sex characteristics, such as facial hair and a deep voice.

For the first 72 hours after thyroidectomy surgery, a nurse should assess a client for Chvostek's sign and Trousseau's sign because they indicate: A. hypokalemia. B. hyperkalemia. C. hypocalcemia. D. hypercalcemia.

C. hypocalcemia. A client who has undergone a thyroidectomy is at risk for developing hypocalcemia from inadvertent removal of or damage to the parathyroid gland. The client with hypocalcemia will exhibit a positive Chvostek's sign (facial muscle contraction when the facial nerve in front of the ear is tapped) and a positive Trousseau's sign (carpal spasm when a blood pressure cuff is inflated for a few minutes). These signs aren't present with hypercalcemia, hypokalemia, or hyperkalemia.

A client with adrenal insufficiency is gravely ill and presents with nausea, vomiting, diarrhea, abdominal pain, profound weakness, and headache. The client's family reports that the client has been doing strenuous yard work all day and was sweating profusely. Nursing management of this client would include observation for signs of: A. hyponatremia and hypokalemia. B. hypernatremia and hypokalemia. C. hyponatremia and hyperkalemia. D. hypernatremia and hyperkalemia.

C. hyponatremia and hyperkalemia. Adrenal crisis may be sudden or gradual. Clients experiencing an adrenal crisis should be monitored for hyponatremia and hyperkalemia.

A client with a history of hypertension is diagnosed with primary hyperaldosteronism. This diagnosis indicates that the client's hypertension is caused by excessive hormone secretion from which gland? A. Pancreas B. Adrenal medulla C. Parathyroid D. Adrenal cortex

D. Adrenal cortex Excessive secretion of aldosterone in the adrenal cortex is responsible for the client's hypertension. This hormone acts on the renal tubule, where it promotes reabsorption of sodium and excretion of potassium and hydrogen ions. The pancreas mainly secretes hormones involved in fuel metabolism. The adrenal medulla secretes the catecholamines — epinephrine and norepinephrine. The parathyroids secrete parathyroid hormone.

Which type of cell secretes glucagon and promotes gluconeogenesis? A. Omega B. Beta C. Delta D. Alpha

D. Alpha The alpha cells of the pancreas secret the hormone glucagon. It promotes gluconeogenesis, thus increasing the blood glucose level. The beta cells of the pancreas secrete insulin. Delta cells secrete somatostatin, which reduces the rate at which food is absorbed from the gastrointestinal tract.

A nurse is caring for a client suspected of having a pituitary tumor that is causing panhypopituitarism. During assessment of the client, which clinical manifestation would the nurse expect to find? A. Tachycardia B. Hypertension C. Carpopedal spasm D. Atrophy of the gonads

D. Atrophy of the gonads Undersecretion (hyposecretion) commonly involves all of the anterior pituitary hormones and is termed panhypopituitarism. In this condition, the thyroid gland, the adrenal cortex, and the gonads atrophy (shrink) because of loss of tropic-stimulating hormones.

Evaluation of an adult client reveals oversecretion of growth hormone. Which of the following would the nurse expect to find? A. Excessive urine output B, Constant thirst C. Weight loss D. Bulging forehead

D. Bulging forehead Oversecretion of growth hormone in an adult results in acromegaly, manifested by coarse features, a huge lower jaw, thick lips, thickened tongue, a bulging forehead, bulbous nose, and large hands and feet. Excessive urine output, weight loss, and constant thirst are associated with diabetes insipidus.

A pheochromocytoma is a rare adrenal tumor that causes increased heart rate, blood pressure, and metabolism because of increased levels of circulating: A. Aldosterone. B. Cortisol. C. Glucocorticoids. D. Catecholamines.

D. Catecholamines. A pheochromocytoma releases high levels of the catecholamines epinephrine and norepinephrine. These levels directly affect the incidence and severity of side effects such as headache, diaphoresis, palpitations, and hypertension. Blood pressure readings exceeding 250/150 have been recorded.

A nurse is planning care for a client in acute addisonian crisis. Which nursing diagnosis should receive the highest priority? A. Imbalanced nutrition: Less than body requirements B. Impaired physical mobility C. Risk for infection D. Decreased cardiac output

D. Decreased cardiac output An acute addisonian crisis is a life-threatening event, caused by deficiencies of cortisol and aldosterone. Glucocorticoid insufficiency causes a decrease in cardiac output and vascular tone, leading to hypovolemia. The client becomes tachycardic and hypotensive and may develop shock and circulatory collapse. The client with Addison's disease is at risk for infection; however, reducing infection isn't a priority during an addisonian crisis. Impaired physical mobility and Imbalanced nutrition: Less than body requirements are appropriate nursing diagnoses for the client with Addison's disease, but they aren't priorities in a crisis.

A client is having chronic pain from arthritis. What type of hormone is released in response to the stress of this pain that suppresses inflammation and helps the body withstand stress? A. Testosterone B. Mineralocorticoids C. Estrogen D. Glucocorticoids

D. Glucocorticoids Glucocorticoids, such as cortisol, affect body metabolism, suppress inflammation, and help the body withstand stress. Mineralocorticoids, primarily aldosterone, maintain water and electrolyte balances. The androgenic hormones convert to testosterone and estrogens.

Which feature(s) indicates a carpopedal spasm in a client with hypoparathyroidism? A. Cardiac dysrhythmia B. Moon face and buffalo hump C. Bulging forehead D. Hand flexing inward

D. Hand flexing inward Carpopedal spasm is evidenced by the hand flexing inward. Cardiac dysrhythmia is a symptom of hyperparathyroidism. Moon face and buffalo hump are the symptoms of Cushing syndrome. A bulging forehead is a symptom of acromegaly.

A client is experiencing an increase in blood glucose levels. The nurse understands that which of the following hormones would be important in lowering the client's blood glucose level? A. Melatonin B. Calcitonin C. Parathormone D. Insulin

D. Insulin Insulin is a hormone released by the beta islet cells that lowers the level of blood glucose when it rises above normal limits. Parathormone increases the level of calcium in the blood when a decrease in serum calcium levels occurs. Melatonin aids in regulating sleep cycles and mood. Calcitonin is a thyroid hormone that inhibits the release of calcium from the bone into the extracellular fluid.

The nurse assesses a patient who has an obvious goiter. What type of deficiency does the nurse recognize is most likely the cause of this? A. Calcitonin B. Thyroxine C. Thyrotropin D. Iodine

D. Iodine Oversecretion of thyroid hormones is usually associated with an enlarged thyroid gland known as a goiter. Goiter also commonly occurs with iodine deficiency.

A patient experiences a life-threatening hypercalcemic crisis. The provider orders a cytotoxic agent. Which of the following is most likely the drug that is prescribed? A. Calcitonin B. Aredia C. Didronel D. Mithramycin

D. Mithramycin Mitramycin is a cytotoxic agent commonly used in a hypercalcemic crisis. Didronel and Aredia are bisphosphonates that decrease serum calcium levels. Calcitonin can be ordered but it is not a cytotoxic agent.

A client with pheochromocytoma is scheduled for an adrenalectomy. Which of the following would the nurse perform preoperatively? A. Initiate intravenous access for fluid therapy. B. Check for the signs of adrenal insufficiency. C. Begin administering prescribed corticosteroid. D. Monitor blood pressure (BP) frequently.

D. Monitor blood pressure (BP) frequently. The nurse should monitor BP frequently before surgery when a client has a pheochromocytoma. When bilateral adrenalectomy is scheduled, the nurse may start IV administration of a solution containing a corticosteroid preparation the morning of surgery. Some surgeons prefer to initiate corticosteroid administration during removal of the adrenals. The nurse monitors for signs of adrenal insufficiency after the surgery.

A client receiving thyroid replacement therapy develops influenza and forgets to take her thyroid replacement medicine. The nurse understands that skipping this medication puts the client at risk for developing which life-threatening complication? A. Exophthalmos B. Tibial myxedema C. Thyroid storm D. Myxedema coma

D. Myxedema coma Myxedema coma, severe hypothyroidism, is a life-threatening condition that may develop if thyroid replacement medication isn't taken. Exophthalmos (protrusion of the eyeballs) is seen with hyperthyroidism. Although thyroid storm is life-threatening, it's caused by severe hyperthyroidism. Tibial myxedema (peripheral mucinous edema involving the lower leg) is associated with hypothyroidism but isn't life-threatening.

Which of the following would the nurse need to be alert for in a client with severe hypothyroidism? A. Acromegaly B. Thyroid storm C. Addison's disease D. Myxedemic coma

D. Myxedemic coma Severe hypothyroidism is called myxedema and if untreated, it can progress to myxedemic coma, a life-threatening event. Thyroid storm is an acute, life-threatening form of hyperthyroidism. Addison's disease refers to primary adrenal insufficiency. Acromegaly refers to an oversecretion of growth hormone by the pituitary gland during adulthood.

Which medication is the treatment of choice for pregnant women diagnosed with hyperthyroidism? A. SSKI B. Methimazole C. Potassium iodide D. PTU

D. PTU Propylthiouracil (PTU), rather than methimazole (MMI), is the treatment of choice during pregnancy for those diagnosed with hyperthyroidism due to the teratogenic effects of MMI.

While assessing a client with hypoparathyroidism, the nurse taps the client's facial nerve and observes twitching of the mouth and tightening of the jaw. The nurse would document this finding as which of the following? A. Positive Trousseau's sign B. Hyperactive deep tendon reflex C. Tetany D. Positive Chvostek's sign

D. Positive Chvostek's sign If a nurse taps the client's facial nerve (which lies under the tissue in front of the ear), the client's mouth twitches and the jaw tightens. The response is identified as a positive Chvostek's sign. The nurse may elicit a positive Trousseau's sign by placing a BP cuff on the upper arm, inflating it between the systolic and diastolic BP, and waiting 3 minutes. The nurse observes the client for spasm of the hand (carpopedal spasm), which is evidenced by the hand flexing inward. Deep tendon reflexes include the biceps, brachioradialis, triceps, and patellar reflexes. Tetany would be manifested by reports of numbness and tingling in the fingers or toes or around the lips, voluntary movement that may be followed by an involuntary, jerking spasm, and muscle cramping. Tonic (continuous contraction) flexion of an arm or a finger may occur.

A nurse is caring for a client in addisonian crisis. Which medication order should the nurse QUESTION? A. Normal saline solution B. Hydrocortisone (Cortef) C. Fludrocortisone (Florinef) D. Potassium chloride

D. Potassium chloride The nurse should question an order for potassium chloride because addisonian crisis results in hyperkalemia. Administering potassium chloride is contraindicated. Because the client is hyponatremic, an order for normal saline solution is appropriate. Hydrocortisone and fludrocortisone are used to replace deficient adrenal cortex hormones.

A client with hyperthyroidism is concerned about changes in appearance. How can the nurse convey an understanding of the client's concern and promote effective coping strategies? A. Refer the client to professional counseling. B. Suggest that the client wear cosmetics to cover any changes in appearance. C. Encourage the client to participate in outside activities to boost coping strategies. D. Reassure the client that their emotional reactions are a result of the disorder and symptoms can be controlled with effective treatment.

D. Reassure the client that their emotional reactions are a result of the disorder and symptoms can be controlled with effective treatment. The client with hyperthyroidism needs reassurance that the emotional reactions being experienced are a result of the disorder and that with effective treatment those symptoms can be controlled. It is important to use a calm, unhurried approach with the client. Stressful experiences should be minimized, and a quiet uncluttered environment should be maintained. The nurse encourages relaxing activities that will not overstimulate the client. It is important to balance periods of activity with rest.

A client with symptoms of Cushing syndrome is admitted to the hospital for evaluation and treatment. The nurse is creating a plan of care for the client. Which is an appropriate nursing diagnosis? A. Impaired nutrition: more than body requirements related to polyphagia B. Activity intolerance related to muscle cramps, cardiac dysrhythmias, and weakness C. Insomnia related to increased nighttime voiding D. Self-care deficit related to weakness, fatigue, muscle wasting, and altered sleep patterns

D. Self-care deficit related to weakness, fatigue, muscle wasting, and altered sleep patterns The major goals for the client include decreased risk of injury, decreased risk of infection, increased ability to perform self-care activities, improved skin integrity, improved body image, improved mental function, and absence of complications. The other nursing diagnoses do not apply in Cushing syndrome.

Patients with hyperthyroidism are characteristically: A. Calm B. Emotionally stable C. Apathetic and anorexic D. Sensitive to heat

D. Sensitive to heat Those with hyperthyroidism tolerate heat poorly and may perspire unusually freely. Their condition is characterized by symptoms of nervousness, hyperexcitability, irritability, and apprehension.

A postpartum client is receiving intravenous oxytocin (Pitocin) after birth. Why will this medication be used for this client after the birth of her child? A. Will prevent lactation for a woman who is bottle feeding her newborn B. Helps treat nausea C. Decreases the postpartum cramping D. Stimulates the contraction of the uterus and prevents bleeding

D. Stimulates the contraction of the uterus and prevents bleeding Oxytocin (Pitocin) is released from the pituitary gland and stimulates contraction of pregnant uterus and release of breast milk after childbirth. It will not prevent lactation or help treat nausea. It will increase lactation.

The nurse is caring for a patient with hyperthyroidism who suddenly develops symptoms related to thyroid storm. What symptoms does the nurse recognize that are indicative of this emergency? A. Blood pressure 90/58 mm Hg B. Heart rate of 62 C. Oxygen saturation of 96% D. Temperature of 102ºF

D. Temperature of 102ºF Thyroid storm is characterized by the following: 1) high fever (hyperpyrexia), >38.5°C (>101.3°F); 2) extreme tachycardia (>130 bpm); 3) exaggerated symptoms of hyperthyroidism with disturbances of a major system—for example, gastrointestinal (weight loss, diarrhea, abdominal pain) or cardiovascular (edema, chest pain, dyspnea, palpitations); and 4) altered neurologic or mental state, which frequently appears as delirium psychosis, somnolence, or coma.

A nurse is assessing a client after a thyroidectomy. The assessment reveals muscle twitching and tingling, along with numbness in the fingers, toes, and mouth area. The nurse should suspect which complication? A. Hemorrhage B. Laryngeal nerve damage C. Thyroid storm D. Tetany

D. Tetany Tetany may result if the parathyroid glands are excised or damaged during thyroid surgery. Hemorrhage is a potential complication after thyroid surgery but is characterized by tachycardia, hypotension, frequent swallowing, feelings of fullness at the incision site, choking, and bleeding. Thyroid storm is another term for severe hyperthyroidism — not a complication of thyroidectomy. Laryngeal nerve damage may occur postoperatively, but its signs include a hoarse voice and, possibly, acute airway obstruction.

When describing the difference between endocrine and exocrine glands, which of the following would the instructor include as characteristic of endocrine glands? A. The glands contain ducts that produce the hormones. B. The secreted hormones act like target cells. C. The glands play a minor role in maintaining homeostasis. D. The secretions are released directly into the blood stream.

D. The secretions are released directly into the blood stream. The endocrine glands secrete hormones, chemicals that accelerate or slow physiologic processes, directly into the bloodstream. This characteristic distinguishes endocrine glands from exocrine glands, which release secretions into a duct. Hormones circulate in the blood until they reach receptors in target cells or other endocrine glands. They play a vital role in regulating homeostatic processes.

What life-threatening outcome should the nurse monitor for in a client who is not compliant with taking his antithyroid medication? A. Syndrome of inappropriate antidiuretic hormone secretion B. Myxedema coma C. Diabetes insipidus D. Thyrotoxic crisis

D. Thyrotoxic crisis Thyrotoxic crisis, an abrupt and life-threatening form of hyperthyroidism, is thought to be triggered by extreme stress, infection, diabetic ketoacidosis, trauma, toxemia of pregnancy, or manipulation of a hyperactive thyroid gland during surgery or physical examination. Although rare, this condition may occur in clients with undiagnosed or inadequately treated hyperthyroidism. Myxedema coma is the opposite in symptoms that thyrotoxic crisis. Diabetes insipidus (DI) and syndrome of inappropriate antidiuretic hormone secretion (SIADH) clinical manifestations do not correlate with medication taken for hyperthyroidism.

A 35-year-old female client who complains of weight gain, facial hair, absent menstruation, frequent bruising, and acne is diagnosed with Cushing's syndrome. Cushing's syndrome is most likely caused by: A. an ectopic corticotropin-secreting tumor. B. an inborn error of metabolism. C. adrenal carcinoma. D. a corticotropin-secreting pituitary adenoma.

D. a corticotropin-secreting pituitary adenoma. A corticotropin-secreting pituitary adenoma is the most common cause of Cushing's syndrome in women ages 20 to 40. Ectopic corticotropin-secreting tumors are more common in older men and are commonly associated with weight loss. Adrenal carcinoma isn't usually accompanied by hirsutism. A female with an inborn error of metabolism wouldn't be menstruating.

A nurse working in the ED at a level 1 trauma center is notified that casualties from a multivehicle car accident are currently in transit. The nurse's heart is pounding and mouth is dry. What gland is responsible for this nurse's physiologic response? A. thyroid gland B. pineal gland C. adrenal cortex D. adrenal medulla

D. adrenal medulla The adrenal medulla secretes epinephrine and norepinephrine. These two hormones are released in response to stress or threat to life. They facilitate what has been referred to as the fight-or-flight response.

Trousseau sign is elicited: A: when the foot is dorsiflexed and there is pain in the calf. B. by tapping sharply over the facial nerve just in front of the parotid gland and anterior to the ear, causing spasm or twitching of the mouth, nose, and eye. C. after making a clenched fist and opening the hand; the palm remains blanched when pressure is placed over the radial artery. D. by occluding the blood flow to the arm for 3 minutes with the use of a blood pressure cuff.

D. by occluding the blood flow to the arm for 3 minutes with the use of a blood pressure cuff. A positive Trousseau sign is suggestive of latent tetany. A positive Chvostek sign is demonstrated when a sharp tapping over the facial nerve just in front of the parotid gland and anterior to the ear causes the mouth, nose, and eye to spasm or twitch. The palm remaining blanched when the radial artery is occluded demonstrates a positive Allen test. The radial artery should not be used for an arterial puncture. A positive Homans sign is demonstrated when the client reports pain in the calf when the foot is dorsiflexed.

A client is being evaluated for hypothyroidism. During assessment, the nurse should stay alert for: A. systolic murmur at the left sternal border. B. exophthalmos and conjunctival redness. C. flushed, warm, moist skin. D. decreased body temperature and cold intolerance.

D. decreased body temperature and cold intolerance. Hypothyroidism markedly decreases the metabolic rate, causing a reduced body temperature and cold intolerance. Other signs and symptoms include dyspnea, hypoventilation, bradycardia, hypotension, anorexia, constipation, decreased intellectual function, and depression. Exophthalmos; conjunctival redness; flushed, warm, moist skin; and a systolic murmur at the left sternal border are typical findings in a client with hyperthyroidism.

When teaching a client with Cushing's syndrome about dietary changes, the nurse should instruct the client to increase intake of: A. dairy products. B. cereals and grains. C. processed meats. D. fresh fruits.

D. fresh fruits. Cushing's syndrome causes sodium retention, which increases urinary potassium loss. Therefore, the nurse should advise the client to increase intake of potassium-rich foods, such as fresh fruit. The client should restrict consumption of dairy products, processed meats, cereals, and grains because they contain significant amounts of sodium.

A client has been experiencing a decrease in serum calcium. After diagnostics, the physician believes the calcium level fluctuation is due to altered parathyroid function. What is the role of parathormone? A. decrease serum calcium level B. inhibit release of calcium into extracellular fluid C. promote urinary secretion of calcium D. increase serum calcium level

D. increase serum calcium level The parathyroid glands secrete parathormone, which increases the level of calcium in the blood when there is a decrease in the serum level.

The nurse is completing discharge teaching with a client with hyperthyroidism who has been treated with radioactive iodine at an outpatient clinic. The nurse instructs the client to A. discontinue all antithyroid medications . B. continue radioactive precautions with all body secretions. C. watch for symptoms of hyperthyroidism to disappear within 1 week. D. monitor for symptoms of hypothyroidism.

D. monitor for symptoms of hypothyroidism. Symptoms of hyperthyroidism may be followed later by those of hypothyroidism and myxedema. Hypothyroidism also commonly occurs in clients with previous hyperthyroidism who have been treated with radioiodine or antithyroid medications or thyroidectomy (surgical removal of all or part of the thyroid gland).

When caring for a client with diabetes insipidus, the nurse expects to administer: A. 10% dextrose. B. regular insulin. C. furosemide. D. vasopressin.

D. vasopressin. Because diabetes insipidus results from decreased antidiuretic hormone (vasopressin) production, the nurse should expect to administer synthetic vasopressin for hormone replacement therapy. Furosemide, a diuretic, is contraindicated because a client with diabetes insipidus experiences polyuria. Insulin and dextrose are used to treat diabetes mellitus and its complications, not diabetes insipidus.

A nurse is instructing a client with newly diagnosed hypoparathyroidism about the regimen used to treat this disorder. The nurse should state that the physician probably will order daily supplements of calcium and: A. iron. B. potassium. C. folic acid. D. vitamin D.

D. vitamin D. Typically, clients with hypoparathyroidism are ordered daily supplements of vitamin D along with calcium because calcium absorption from the small intestine depends on vitamin D. Hypoparathyroidism doesn't cause a deficiency of folic acid, potassium, or iron. Therefore, the client doesn't require daily supplements of these substances to maintain a normal serum calcium level.

The nurse practitioner who assesses a patient with hyperthyroidism would expect the patient to report which of the following conditions? A. Dyspnea B. Hair loss C. Weight loss D. Fatigue

C. Weight loss Weight loss is consistent with a diagnosis of hyperthyroidism. The other conditions are found in hypothyroidism.

A client with severe hypoparathyroidism is experiencing tetany. What medication, prescribed by the physician for emergency use, will the nurse administer to correct the deficit?

Calcium gluconate

A client requires hemodialysis. Which type of drug should be withheld before this procedure?

Cardiac glycosides

Patient education regarding a fistulae or graft includes which of the following? Select all that apply.

Check daily for thrill and bruit. Avoid compression of the site. No IV or blood pressure taken on extremity with dialysis access. No tight clothing.

A client who agreed to become an organ donor is pronounced dead. What is the most important factor in selecting a transplant recipient?

Compatible blood and tissue types

A patient has been diagnosed with syndrome of inappropriate antidiuretic hormone secretion (SIADH). Which of the following manifestations would be expected in this patient?

Concentrated urine

What interventions can the nurse encourage the client with diabetes insipidus to do in order to control thirst and compensate for urine loss?

Consume adequate amounts of fluid.

Hypophysectomy is the treatment of choice for which endocrine disorder?

Cushing syndrome

A client visits the physician's office complaining of agitation, restlessness, and weight loss. The physical examination reveals exophthalmos, a classic sign of Graves' disease. Based on history and physical findings, the nurse suspects hyperthyroidism. Exophthalmos is characterized by: A. dry, waxy swelling and abnormal mucin deposits in the skin. B. a wide, staggering gait. C. more than 10 beats/minute difference between the apical and radial pulse rates. D. protruding eyes and a fixed stare.

D. protruding eyes and a fixed stare. Exophthalmos is characterized by protruding eyes and a fixed stare. Dry, waxy swelling and abnormal mucin deposits in the skin typify myxedema, a condition resulting from advanced hypothyroidism. A wide, staggering gait and a differential between the apical and radial pulse rates aren't specific signs of thyroid dysfunction.

A patient is having diagnostic testing for suspected hyperthyroidism. Which of the following diagnostics correlate with this endocrine disorder? Select all that apply.

Decrease in serum thyroid-stimulating hormone (TSH) Increased T3 Increased T4 Increase in radioactive iodine uptake

A nurse caring for a client with diabetes insipidus is reviewing laboratory results. What is an expected urinalysis finding?

Highly dilute urine

A client with a history of Addison's disease and flu-like symptoms accompanied by nausea and vomiting over the past week is brought to the facility. His wife reports that he acted confused and was extremely weak when he awoke that morning. The client's blood pressure is 90/58 mm Hg, his pulse is 116 beats/minute, and his temperature is 101° F (38.3° C). A diagnosis of acute adrenal insufficiency is made. What should the nurse expect to administer by IV infusion?

Hydrocortisone

The nurse's assessment of a client with thyroidectomy suggests tetany and a review of the most recent blood work corroborates this finding. The nurse should prepare to administer what intervention?

IV calcium gluconate

Acute dialysis is indicated during which situation?

Impending pulmonary edema

A nurse teaches a client with newly diagnosed hypothyroidism about the need for thyroid hormone replacement therapy to restore normal thyroid function. Which thyroid preparation is the agent of choice for thyroid hormone replacement therapy?

Levothyroxine (Synthroid)

The nurse is caring for a client with hyperparathyroidism. What level of activity would the nurse expect to promote?

Mobility, with walking or use of a rocking chair for those with limited mobility, is encouraged as much as possible because bones subjected to normal stress give up less calcium. Bed rest should be discouraged because it increases calcium excretion and the risk of renal calculi. Limiting the client to getting out of bed only a few times a day also increases calcium excretion and the associated risks.

The nurse is caring for a client with a diagnosis of Addison disease. What sign or symptom is most closely associated with this health problem?

Muscle weakness

During hemodialysis, excess water is removed from the blood by which of the following?

Osmosis

The nurse is caring for a client with hypoparathyroidism. When the nurse taps the client's facial nerve, the client's mouth twitches and the jaw tightens. What is this response documented as related to the low calcium levels?

Positive Chvostek's sign

The nurse is teaching a client that the body needs iodine for the thyroid to function. What food would be the best source of iodine for the body?

Table salt

A client with chronic kidney disease is completing an exchange during peritoneal dialysis. The nurse observes that the peritoneal fluid is draining slowly and that the client's abdomen is increasing in girth. What is the nurse's most appropriate action?

Reposition the client to facilitate drainage.

The nurse recognizes that which of the following agents suppress release of thyroid hormones? Select all that apply.

Sodium iodide Saturated solution of potassium iodide (SSKI) Potassium iodide

A client has undergone a renal transplant and returns to the health care agency for a follow-up evaluation. Which finding would lead to the suspicion that the client is experiencing rejection?

Tenderness over transplant site

Nursing care for a client in addisonian crisis should include which intervention?

The client in addisonian crisis has a reduced ability to cope with stress as a result of an inability to produce corticosteroids. A private room is easy to keep quiet, dimly lit, and temperature controlled. Also, visitors can be limited to reduce noise, promote rest, and decrease the risk of infection. The client should be kept on bed rest, receiving total assistance with ADLs to avoid stress as much as possible. Because extremes of temperature should be avoided, measures to raise the body temperature, such as extra blankets and turning up the heat, should be avoided.

The nurse is caring for a client with Addison disease who is scheduled for discharge. When teaching the client about hormone replacement therapy, the nurse should address what topic?

The need for lifelong steroid replacement

A patient is placed on hemodialysis for the first time. The patient complains of a headache with nausea and begins to vomit, and the nurse observes a decreased level of consciousness. What does the nurse determine has happened?

The patient is experiencing a cerebral fluid shift.

he nurse is working on the renal transplant unit. To reduce the risk of infection in a client with a transplanted kidney, it is imperative for the nurse to do what?

Wash hands carefully and frequently.

A male client has doubts about performing peritoneal dialysis at home. He informs the nurse about his existing upper respiratory infection. Which of the following suggestions can the nurse offer to the client while performing an at-home peritoneal dialysis?

Wear a mask when performing exchanges.

During an assessment of a client's functional health pattern, which question by the nurse directly addresses the client's thyroid function? A. "Do you experience fatigue even if you have slept a long time?" B. "Do you have to get up at night to empty your bladder?" C. "Have you experienced any headaches or sinus problems?" D. "Can you describe the amount of stress in your life?"

A. "Do you experience fatigue even if you have slept a long time?" With the diagnosis of hypothyroidism, extreme fatigue makes it difficult for the person to complete a full day's work or participate in usual activities.

A group of students are reviewing information about the relationship of the hypothalamus and the pituitary gland. The students demonstrate the need for additional study when they state which of the following? A. "The pituitary gland, as the master gland, controls the secretion of hormones by the hypothalamus." B. "The hypothalamus secretes releasing hormones that stimulate or inhibit pituitary gland secretions." C. "The hypothalamus, a portion of the brain between the cerebrum and brain stem, creates a pathway for neurohormones." D. "Corticotropin-releasing hormone from the hypothalamus triggers ACTH secretion by the pituitary gland."

A. "The pituitary gland, as the master gland, controls the secretion of hormones by the hypothalamus." Although the pituitary gland is considered the master gland because it regulates the function of other glands, the hypothalamus influences the pituitary gland. The hypothalamus creates a pathway for neurohormones also known as releasing hormones or factors that stimulate and inhibit secretions from the anterior and posterior lobes of the pituitary gland. Under the influence of the hypothalamus, the lobes of the pituitary gland secrete various hormones. For example, corticotropin-releasing hormone from the hypothalamus causes the anterior pituitary gland to secreted ACTH.

During preoperative teaching for a client who will undergo subtotal thyroidectomy, the nurse should include which statement? A. "You must avoid hyperextending your neck after surgery." B. "The head of your bed must remain flat for 24 hours after surgery." C. "You should avoid deep breathing and coughing after surgery." D. "You won't be able to swallow for the first day or two."

A. "You must avoid hyperextending your neck after surgery." To prevent undue pressure on the surgical incision after subtotal thyroidectomy, the nurse should advise the client to avoid hyperextending the neck. The client may elevate the head of the bed as desired and should perform deep breathing and coughing to help prevent pneumonia. Subtotal thyroidectomy doesn't affect swallowing.

Which condition should a nurse expect to find in a client diagnosed with hyperparathyroidism? A. Hypercalcemia B. Hyperphosphatemia C. Hypocalcemia D. Hypophosphaturia

A. Hypercalcemia Hypercalcemia is the hallmark of excess parathyroid hormone levels. Serum phosphate will be low (hyperphosphatemia), and there will be increased urinary phosphate (hyperphosphaturia) because phosphate excretion is increased.

A nurse teaches a client with newly diagnosed hypothyroidism about the need for thyroid hormone replacement therapy to restore normal thyroid function. Which thyroid preparation is the agent of choice for thyroid hormone replacement therapy? A. Levothyroxine (Synthroid) B. Thyroid USP desiccated (Thyroid USP Enseals) C. Methimazole (Tapazole) D. Liothyronine (Cytomel)

A. Levothyroxine (Synthroid) Levothyroxine is the agent of choice for thyroid hormone replacement therapy because its standard hormone content provides predictable results. Methimazole is an antithyroid medication used to treat hyperthyroidism. Thyroid USP desiccated and liothyronine are no longer used for thyroid hormone replacement therapy because they may cause fluctuating plasma drug levels, increasing the risk of adverse effects.

A client is suspected to have a pituitary tumor due to signs of diabetes insipidus. What initial test does the nurse help to prepare the client for? A. Magnetic resonance imaging (MRI) B. Radioimmunoassay C. A nuclear scan D. Radioactive iodine uptake test

A. Magnetic resonance imaging (MRI) A computed tomography (CT) or magnetic resonance imaging (MRI) scan is performed to detect a suspected pituitary tumor or to identify calcifications or tumors of the parathyroid glands. A radioactive iodine uptake test would be useful for a thyroid tumor. Radioimmunoassay determines the concentration of a substance in plasma.

A nurse is caring for a client who had a thyroidectomy and is at risk for hypocalcemia. What should the nurse do? A. Observe for muscle twitching and numbness or tingling of the lips, fingers, and toes. B. Evaluate the quality of the client's voice postoperatively, noting any drastic changes. C. Monitor laboratory values daily for elevated thyroid-stimulating hormone. D. Observe for swelling of the neck, tracheal deviation, and severe pain.

A. Observe for muscle twitching and numbness or tingling of the lips, fingers, and toes. Muscle twitching and numbness or tingling of the lips, fingers, and toes are signs of hyperirritability of the nervous system due to hypocalcemia. The other options describe complications for which the nurse should also be observing; however, tetany and neurologic alterations are primary indications of hypocalcemia.

A client is undergoing a diagnostic workup for suspected thyroid cancer. What is the most common form of thyroid cancer in adults? A. Papillary carcinoma B. Follicular carcinoma C. Anaplastic carcinoma D. Medullary carcinoma

A. Papillary carcinoma Papillary carcinoma accounts for about 70% of thyroid cancer cases in adults. Follicular carcinoma accounts for roughly 15%; anaplastic carcinoma, about 5%; and medullary carcinoma, about 5%.

On assessment of a patient with early-stage hypothyroidism, the nurse practitioner assesses for a vague yet significant sign which is: A. Paresthesia B. Hypotension C. Bradypnea D. Hypothermia

A. Paresthesia Paresthesia refers to numbness and tingling of the fingers. It is a vague sign that is frequently ignored, yet it is linked with hypothyroidism.

A client who is frightened of needles has been told that he will have to have an intravenous (IV) line inserted. The client's blood pressure and pulse rate increase, and the nurse observes the pupils dilating. What does the nurse recognize has occurred with this client? A. The client is showing the fight-or-flight response. B. The client is having a response to dehydration. C. The client is developing an infection. D. The client is in a hypertensive crisis.

A. The client is showing the fight-or-flight response. The adrenal medulla secretes epinephrine and norepinephrine. These two hormones are released in response to stress or threat to life. They facilitate what is referred to as the physiologic stress response, also known as the fight-or-flight response. Many organs respond to the release of epinephrine and norepinephrine. Responses include increased blood pressure and pulse rate, dilation of the pupils, constriction of blood vessels, bronchodilation, and decreased peristalsis. The client does not demonstrate the signs of infection, dehydration, or hypertensive crisis.

Which group of clients should not receive potassium iodide? A. Those who are allergic to seafood B. Those who are pregnant C. Those who are allergic to corticosteroids D. Those taking medications such as cough medicines

A. Those who are allergic to seafood Potassium iodide should not be administered to anyone who is allergic to seafood, which is also high in iodine. Clients who take corticosteroids or cough medicines and those who are pregnant would be appropriate candidates for potassium iodide therapy.

A nurse is caring for a client with syndrome of inappropriate antidiuretic hormone secretion (SIADH). Which finding would indicate that the client has developed fluid overload? A. dyspnea and hypertension B. confusion and diarrhea C. hypertension and weight gain without edema D. pulmonary congestion and muscle cramps

A. dyspnea and hypertension Signs of fluid overload would include confusion, dyspnea, pulmonary congestion, and hypertension. Muscle cramps, diarrhea, and weight gain without edema would be indicative of hyponatremia.

A nurse is assessing a client with Cushing's syndrome. Which observation should the nurse report to the physician immediately?

An irregular apical pulse

The nurse passes out medications while a client prepares for hemodialysis. The client is ordered to receive numerous medications including antihypertensives. What is the best action for the nurse to take?

Antihypertensive therapy, often part of the regimen of clients on dialysis, is one example when communication, education, and evaluation can make a difference in client outcomes. The client must know when—and when not—to take the medication. For example, if an antihypertensive agent is taken on a dialysis day, hypotension may occur during dialysis, causing dangerously low blood pressure. Many medications that are taken once daily can be held until after dialysis treatment.

A client with chronic kidney disease has been hospitalized and is receiving hemodialysis on a scheduled basis. The nurse should include which of the following actions in the plan of care?

Assess for a thrill or bruit over the vascular access site each shift

A client with a history of chronic renal failure receives hemodialysis treatments three times per week through an arteriovenous (AV) fistula in the left arm. Which intervention should the nurse include in the care plan?

Assess the AV fistula for a bruit and thrill.

During the first 24 hours after a client is diagnosed with addisonian crisis, which intervention should the nurse perform frequently?

Assess vital signs.

What is the term for a concentration of urea and other nitrogenous wastes in the blood?

Azotemia

A client with hyperparathyroidism declines surgery and is to receive hormone replacement therapy with estrogen and progesterone. Which instruction is most important for the nurse to include in the client's teaching plan? A. "Jog at least 2 miles per day." B. "Maintain a moderate exercise program." C. "Rest as much as possible." D. "Lose weight."

B. "Maintain a moderate exercise program." The nurse should instruct the client to maintain a moderate exercise program. Such a program helps strengthen bones and prevents the bone loss that occurs from excess parathyroid hormone. Walking or swimming provides the most beneficial exercise. Because of weakened bones, a rigorous exercise program such as jogging is contraindicated. Weight loss might be beneficial but it isn't as important as developing a moderate exercise program.

A nurse is assessing a client with Cushing's syndrome. Which observation should the nurse report to the physician immediately? A. Dry mucous membranes B. An irregular apical pulse

B. An irregular apical pulse Because Cushing's syndrome causes aldosterone overproduction, which increases urinary potassium loss, the disorder may lead to hypokalemia. Therefore, the nurse should immediately report signs and symptoms of hypokalemia, such as an irregular apical pulse, to the physician. Edema is an expected finding because aldosterone overproduction causes sodium and fluid retention. Dry mucous membranes and frequent urination signal dehydration, which isn't associated with Cushing's syndrome.

A client has suffered from several autoimmune disorders over the last 25 years, and lately has developed a new set of symptoms. The client's healthcare provider suspects Addison's disease. Which symptom would the nurse NOT expect to see? A. depression B. weight gain C. hypoglycemia D. hypotension

B. weight gain Weight loss, anemia, anorexia, and gastrointestinal symptoms are signs and symptoms of adrenal insufficiency. The consequences of decreased adrenal cortical function include decreased available glucose and hypoglycemia. Nervousness and periods of depression are often seen in clients with adrenal insufficiency. Weakness, fatigue, dizziness, hypotension, postural hypotension, and hypothermia are often seen in clients with adrenal insufficiency.

The nurse is assessing a client diagnosed with Graves disease. What physical characteristics of Graves disease would the nurse expect to find?

Bulging eyes

Which disorder results from excessive secretion of somatotropin (growth hormone)? A. Cretinism B. Adrenogenital syndrome C. Acromegaly D. Dwarfism

C. Acromegaly The client with acromegaly demonstrates progressive enlargement of peripheral body parts, most commonly the face, head, hands, and feet. Cretinism occurs as a result of congenital hypothyroidism. Dwarfism is caused by insufficient secretion of growth hormone during childhood. Adrenogenital syndrome is the result of abnormal secretion of adrenocortical hormones, especially androgen.

A health care provider suspects that a thyroid nodule may be malignant. The nurse knows to prepare information for the patient based on the usual test that will be ordered to establish a diagnosis. What is that test? A. Ultrasound of the thyroid gland B. Serum immunoassay for TSH C. Fine-needle biopsy of the thyroid gland D. Free T4 analysis

C. Fine-needle biopsy of the thyroid gland Fine needle biopsy of the thyroid gland is often used to establish the diagnosis of thyroid cancer. The purpose of the biopsy is to differentiate cancerous thyroid nodules from noncancerous nodules and to stage the cancer if detected. The procedure is safe and usually requires only a local anesthetic.

Which of the following clinical signs are associated with diabetes insipidus? A. Hypertension B. Oliguria C. Hypotension D. Bradycardia

C. Hypotension Diabetes insipidus, which causes profound polyuria, may cause clinical signs of volume depletion such as tachycardia and hypotension.

One of the most frequently occurring complications (55% occurrence) of primary hyperparathyroidism is: A. Pathologic fractures. B. Peptic ulcer. C. Kidney stones. D. Pancreatitis.

C. Kidney stones. Kidney stones occur in 55% of patients with primary hyperparathyroidism. They are caused by renal damage from the precipitation of calcium phosphate in the renal pelvis and parenchyma.

The nurse is reviewing the history and physical examination of a client diagnosed with hyperthyroidism. Which of the following would the nurse expect to find? A. Thick hard nails B. Inability to tolerate cold C. Reports of increased appetite D. Complaints of sleepiness

C. Reports of increased appetite Signs and symptoms of hyperthyroidism reflect the increased metabolic rate and would include reports of increased appetite, weight loss, and intolerance to heat. Sleepiness, thick hard nails, and intolerance to cold are associated with hypothyroidism.

Dilutional hyponatremia occurs in which disorder? A. Addison disease B. Diabetes insipidus (DI) C. Syndrome of inappropriate antidiuretic hormone secretion (SIADH) D. Pheochromocytoma

C. Syndrome of inappropriate antidiuretic hormone secretion (SIADH) Clients diagnosed with SIADH exhibit dilutional hyponatremia. They retain fluids and develop a sodium deficiency.

A client presents with a huge lower jaw, bulging forehead, large hands and feet, and frequent headaches. What could be causing this client's symptoms? A. hypopituitarism B. panhypopituitarism C. hyperpituitarism D. panhyperpituitarism

C. hyperpituitarism Acromegaly (hyperpituitarism) is a condition in which growth hormone is oversecreted after the epiphyses of the long bones have sealed. A client with acromegaly has coarse features, a huge lower jaw, thick lips, a thickened tongue, a bulging forehead, a bulbous nose, and large hands and feet. When the overgrowth is from a tumor, headaches caused by pressure on the sella turcica are common.

The actions of parathyroid hormone (PTH) are increased in the presence of which vitamin? A. C B. E C. B D. D

D. D The actions of PTH are increased by the presence of vitamin D.

A nurse is performing an examination and notes that the client exhibits signs of exophthalmos. What has the nurse observed? A. enlarged thyroid gland B. excessive hair growth C. changes in pigmentation D. abnormal bulging or protrusion of the eyes

D. abnormal bulging or protrusion of the eyes When there is an increase in the volume of the tissue behind the eyes, the eyes will appear to bulge out of the face. Exophthalmos is a bulging of the eye anteriorly out of the orbit.

Hyperthyroidism is caused by increased levels of thyroxine in blood plasma. A client with this endocrine dysfunction experiences: A. weight gain and heat intolerance. B. diastolic hypertension and widened pulse pressure. C. anorexia and hyperexcitability. D. heat intolerance and systolic hypertension.

D. heat intolerance and systolic hypertension. An increased metabolic rate in a client with hyperthyroidism caused by excess serum thyroxine leads to systolic hypertension and heat intolerance. Weight loss — not gain — occurs because of the increased metabolic rate. Diastolic blood pressure decreases because of decreased peripheral resistance. Heat intolerance and widened pulse pressure can occur but systolic hypertension and diastolic hypertension don't. Clients with hyperthyroidism experience an increase in appetite — not anorexia.

Before undergoing a subtotal thyroidectomy, a client receives potassium iodide (Lugol's solution) and propylthiouracil (PTU). The nurse should expect the client's symptoms to subside: A. immediately. B. in 3 to 4 months. C. in a few days. D. in 1 to 2 weeks.

D. in 1 to 2 weeks. Potassium iodide reduces the vascularity of the thyroid gland and is used to prepare the gland for surgery. Potassium iodide reaches its maximum effect in 1 to 2 weeks. PTU blocks the conversion of thyroxine to triiodothyronine, the more biologically active thyroid hormone. PTU effects are also seen in 1 to 2 weeks. To relieve symptoms of hyperthyroidism in the interim, clients are usually given a beta-adrenergic blocker such as propranolol (Inderal).

Which of the following medications is used in the treatment of diabetes insipidus to control fluid balance?

Desmopressin (DDAVP)

A client receiving thyroid replacement therapy develops influenza and forgets to take her thyroid replacement medicine. The nurse understands that skipping this medication puts the client at risk for developing which life-threatening complication?

Myxedema coma

A nurse is caring for a client with syndrome of inappropriate antidiuretic hormone secretion (SIADH). Which finding would indicate that the client has developed fluid overload?

dyspnea and hypertension

Which are correct statements about the relationship between the hypothalamus and the pituitary gland? Select all that apply.

Under the influence of the hypothalamus, the lobes of the pituitary gland secrete various hormones. Many endocrine glands respond to stimulation from the pituitary gland, which is connected by a stalk to the hypothalamus in the brain.

The nurse knows to assess a patient with hyperthyroidism for the primary indicator of: A. Constipation B. Weight gain C. Intolerance to heat D. Fatigue

C. Intolerance to heat With hypothyroidism, the individual is sensitive to cold because the core body temperature is usually below 98.6°F. Intolerance to heat is seen with hyperthyroidism. Not fatigue because fatigue is not a primary indicator.

When assessing a client with pheochromocytoma, a tumor of the adrenal medulla that secretes excessive catecholamine, the nurse is most likely to detect: A. a blood pressure of 130/70 mm Hg. B. bradycardia. C. a blood pressure of 176/88 mm Hg. D. a blood glucose level of 130 mg/dl.

C. a blood pressure of 176/88 mm Hg. Pheochromocytoma causes hypertension, tachycardia, hyperglycemia, hypermetabolism, and weight loss. It isn't associated with hypotension, hypoglycemia, or bradycardia.

A nurse receives her client care assignment. Following the report, she should give priority assessment to the client:

who, following a kidney transplant, has returned from hemodialysis with a sodium level of 110 mEq/L and a potassium level of 2.0 mEq/L.

What dietary modifications should be recommended to a client with hyperthyroidism? A. Consume a high-protein diet. B. Restrict calorie intake. C. Increase calorie intake by 70%. D. Limit intake of nutritionally dense foods such as milk products, eggs, and cheese.

A. Consume a high-protein diet. A high protein intake helps replenish losses from muscle catabolism. Metabolism is increased with hyperthyroidism. Calorie needs increase between 10% and 50% above normal to replenish glycogen stores and correct weight loss. Encourage frequent meals and the intake of nutritionally dense foods (fortified milkshakes, foods fortified with skim milk powder, eggs, cheese, butter, or milk).

At the end of five peritoneal exchanges, a patient's fluid loss was 500 mL. How much is this loss equal to?

1.0 lb

A dialysis client is prescribed erythropoietin (Epogen) to treat anemia associated with end-stage renal disease. The client weighs 147 lbs. The order is for Epogen 50 units/kg subcutaneously 3 times per week. The pharmacy supplied Epogen 3000 units/ml. How many milliliters will the nurse administer to the client? Round to the nearest tenth.

1.1

The nurse is teaching a client about the dietary restrictions related to his diagnosis of hyperparathyroidism. What foods should the nurse encourage the client to avoid? A. Milk B. Hamburger C. Bananas D. Chicken livers

A. Milk Clients with hyperparathyroidism should use a low-calcium diet (fewer dairy products) and drink at least 3 to 4 L of fluid daily to dilute the urine and prevent renal stones from forming. It is especially important that the client drink fluids before going to bed and periodically throughout the night to avoid concentrated urine. Bananas, chicken livers, and hamburgers do not require avoidance. Milk is the highest in calcium content.

A nurse is caring for a client who was recently diagnosed with hyperparathyroidism. Which statement by the client indicates the need for additional discharge teaching? A. "I will increase my fluid and calcium intake." B. "I will take my pain medications according to the schedule we developed." C. "I'll call my physician if I notice tingling around my lips." D. "I'll schedule a follow-up visit with my physician as soon as I get home."

A. "I will increase my fluid and calcium intake." The client requires additional teaching if he states that he will increase his calcium intake. Hyperparathyroidism causes extreme increases in serum calcium levels. The client should increase his fluid intake, but he should limit his calcium and vitamin D intake. The client should continue to take pain mediations as scheduled and have regular follow-up visits with his physician. Tingling around the lips is a sign of hypercalcemia and should be reported to the physician immediately.

A nurse is teaching a client recovering from addisonian crisis about the need to take fludrocortisone acetate and hydrocortisone at home. Which statement by the client indicates an understanding of the instructions? A. "I'll take two-thirds of the dose when I wake up and one-third in the late afternoon." B. "I'll take my hydrocortisone in the late afternoon, before dinner." C. "I'll take the entire dose at bedtime." D. "I'll take all of my hydrocortisone in the morning, right after I wake up."

A. "I'll take two-thirds of the dose when I wake up and one-third in the late afternoon." Hydrocortisone, a glucocorticoid, should be administered according to a schedule that closely reflects the body's own secretion of this hormone; therefore, two-thirds of the dose of hydrocortisone should be taken in the morning and one-third in the late afternoon. This dosage schedule reduces adverse effects.

Which of the following is the primary hormone for the long-term regulation of sodium balance? A. Aldosterone B. Calcitonin C. Antidiuretic hormone (ADH) D. Thyroxin

A. Aldosterone aldosterone is the primary hormone for the long-term regulation of sodium balance. Vasopressin (ADH) release will result in reabsorption of water into the bloodstream, rather than excretion by the kidneys. Calcitonin is secreted in response to high plasma levels of calcium, and it reduces the plasma level of calcium by increasing its deposition in bone. Thyroxin is important in brain development and is necessary for normal growth.

Hypocalcemia is associated with which of the following manifestations? A. Muscle twitching B. Bowel hypomotility C. Fatigue D. Polyuria

A. Muscle twitching Clinical manifestations of hypocalcemia include paresthesias and fasciculations (muscle twitching).

A client with Cushing syndrome is admitted to the hospital. During the initial assessment, the client tells the nurse, "The worst thing about this disease is how awful I look. I want to cry every time I look in the mirror." Which statements by the nurse is the best response? A. "I can show you how to change your style of dress so that the changes are not so noticeable." B. "If treated successfully, the major physical changes will disappear with time." C. "I do not think you look bad and I am sure your family loves you very much." D. "I can refer you to a support group. Talking to someone may help you feel better."

B. "If treated successfully, the major physical changes will disappear with time." If treated successfully, the major physical changes associated with Cushing syndrome disappear with time. The client may benefit from discussion of the effect the changes have had on his or her self-concept and relationships with others. Weight gain and edema may be modified by a low-carbohydrate, low-sodium diet, and a high protein intake may reduce some of the other bothersome symptoms.

Accidental removal of one or both parathyroid glands can occur during a thyroidectomy. Which of the following is used to treat tetany? A. Propylthiouracil (PTU) B. Calcium gluconate C. Synthroid D. Tapazole

B. Calcium gluconate Sometimes in thyroid surgery, the parathyroid glands are removed, producing a disturbance in calcium metabolism. Tetany is usually treated with IV calcium gluconate. Synthroid is used in the treatment of hypothyroidism. PTU and Tapazole are used in the treatment of hyperthyroidism.

What interventions can the nurse encourage the client with diabetes insipidus to do in order to control thirst and compensate for urine loss? A. Limit the fluid intake at night. B. Consume adequate amounts of fluid. C. Come to the clinic for IV fluid therapy daily. D. Weigh daily.

B. Consume adequate amounts of fluid. The nurse teaches the client to consume sufficient fluid to control thirst and to compensate for urine loss. The client will not be required to come in daily for IV fluid therapy. The client should not limit fluid intake at night if thirst is present. Weighing daily will not control thirst or compensate for urine loss.

Undersecretion of thyroid hormone during fetal and neonatal development can cause which of the following? A. Myxedema B. Cretinism C. Diabetes insipidus D. Hypothyroidism

B. Cretinism During fetal and neonatal development, undersecretion of thyroid hormone may cause cretinism (stunted growth and mental development). In adults, hyposecretion of thyroid hormone causes myxedema or hypothyroidism. Diabetes insipidus is caused by undersecretion of antidiuretic hormone (ADH/vasopressin).

Which disorder is characterized by a group of symptoms produced by an excess of free circulating cortisol from the adrenal cortex? A. Hashimoto disease B. Cushing syndrome C. Graves disease D. Addison disease

B. Cushing syndrome The client with Cushing syndrome demonstrates truncal obesity, moon face, acne, abdominal striae, and hypertension. Regardless of the cause, the normal feedback mechanisms that control the function of the adrenal cortex become ineffective, and the usual diurnal pattern of cortisol is lost. The signs and symptoms of Cushing syndrome are primarily a result of the oversecretion of glucocorticoids and androgens, although mineralocorticoid secretion also may be affected.

A client is admitted for treatment of the syndrome of inappropriate antidiuretic hormone (SIADH). Which nursing intervention is appropriate? A. Encouraging increased oral intake B. Restricting fluids C. Administering glucose-containing I.V. fluids as ordered D. Infusing IV fluids rapidly as ordered

B. Restricting fluids To reduce water retention in a client with the SIADH, the nurse should restrict fluids. Administering fluids by any route would further increase the client's already heightened fluid load.

When caring for a client who's being treated for hyperthyroidism, the nurse should: A. monitor the client for signs of restlessness, sweating, and excessive weight loss during thyroid replacement therapy. B. balance the client's periods of activity and rest. C. encourage the client to be active to prevent constipation. D. provide extra blankets and clothing to keep the client warm.

B. balance the client's periods of activity and rest. A client with hyperthyroidism needs to be encouraged to balance periods of activity and rest. Many clients with hyperthyroidism are hyperactive and complain of feeling very warm. Consequently, it's important to keep the environment cool and to teach the client how to manage his physical reactions to heat. Clients with hypothyroidism — not hyperthyroidism — complain of being cold and need warm clothing and blankets to maintain a comfortable temperature. They also receive thyroid replacement therapy, commonly feel lethargic and sluggish, and are prone to constipation. The nurse should encourage clients with hypothyroidism to be more active to prevent constipation.

A nurse is assessing a client with possible Cushing's syndrome. In a client with Cushing's syndrome, the nurse expects to find: A. weight gain in arms and legs. B. deposits of adipose tissue in the trunk and dorsocervical area. C. hypotension. D. thick, coarse skin.

B. deposits of adipose tissue in the trunk and dorsocervical area. Because of changes in fat distribution, adipose tissue accumulates in the trunk, face (moon face), and dorsocervical areas (buffalo hump). Hypertension is caused by fluid retention. Skin becomes thin and bruises easily because of a loss of collagen. Muscle wasting causes muscle atrophy and thin extremities.

During a follow-up visit to the physician, a client with hyperparathyroidism asks the nurse to explain the physiology of the parathyroid glands. The nurse states that these glands produce parathyroid hormone (PTH). PTH maintains the balance between calcium and: A. magnesium. B. phosphorus. C. potassium. D. sodium.

B. phosphorus. PTH increases the serum calcium level and decreases the serum phosphate level. PTH doesn't affect sodium, potassium, or magnesium regulation.

A nurse should expect a client with hypothyroidism to report: A. nervousness and tremors. B. puffiness of the face and hands. C. increased appetite and weight loss. D. thyroid gland swelling.

B. puffiness of the face and hands. Hypothyroidism (myxedema) causes facial puffiness, extremity edema, and weight gain. Signs and symptoms of hyperthyroidism (Graves' disease) include an increased appetite, weight loss, nervousness, tremors, and thyroid gland enlargement (goiter).

A nurse is collecting data from a client who is suspected of having an endocrine disorder and is scheduled for diagnostic testing that involves the use of a contrast medium. The nurse would inform the physician if the client stated which of the following? A. "My father had diabetes when he got older." B. "Years ago, I took a steroid for my asthma." C. "I have an allergy to shrimp and shellfish." D. "I take a multivitamin every day."

C. "I have an allergy to shrimp and shellfish." The statement about an allergy to shrimp and shellfish would alert the nurse to the possibility of an allergy to iodine, a component of contrast media. This information needs to be reported to the physician. Although information about a family history of diabetes would be important, it would have no effect on the diagnostic testing. Certain drugs can affect diagnostic testing, but a multivitamin probably would not be a problem. Use of steroids within the past 3 months, rather than several years ago, could impact the results of thyroid testing.

A client who is being tested for syndrome of inappropriate antidiuretic hormone secretion asks the nurse to explain the diagnosis. While explaining, the nurse states that excessive antidiuretic hormone is secreted from which gland? A. Thyroid B. Adrenal C. Posterior pituitary D. Anterior pituitary

C. Posterior pituitary Antidiuretic hormone is secreted by the posterior pituitary gland.

A nurse is caring for a client with Cushing's syndrome. Which would the nurse not include in this client's plan of care? A. Examine extremities for pitting edema. B. Report systolic BP that exceeds 139 mm Hg or diastolic BP that exceeds 89 mm Hg. C. Provide a high-sodium diet. D.Administer prescribed diuretics.

C. Provide a high-sodium diet. Limiting sodium reduces the potential for fluid retention. Fluid retention is manifested by swelling in dependent areas, pitting when pressure is applied to the skin over a bone by tight-fitting shoes or rings, the appearance of lines in the skin from stockings and seams in the shoes or areas where they lace. Hypertension is defined as a consistently elevated BP above 139/89 mm Hg. One factor that contributes to hypertension is excess circulatory volume. Diuretics promote the excretion of sodium and water.

A nurse explains to a client with thyroid disease that the thyroid gland normally produces: A. thyrotropin-releasing hormone (TRH) and TSH. B. TSH, triiodothyronine (T3), and calcitonin. C. T3, thyroxine (T4), and calcitonin. D. iodine and thyroid-stimulating hormone (TSH).

C. T3, thyroxine (T4), and calcitonin. The thyroid gland normally produces thyroid hormone (T3 and T4) and calcitonin. The pituitary gland produces TSH to regulate the thyroid gland. The hypothalamus gland produces TRH to regulate the pituitary gland.

A nursing student asks the instructor why the pituitary gland is called the "master gland." What is the best response by the instructor? A. "The gland does not have any other function other than to cause secretion of the growth hormones." B. "It is the gland that is responsible for regulating the hypothalamus." C. "It regulates metabolism." D. "It regulates the function of other endocrine glands."

D. "It regulates the function of other endocrine glands." The pituitary gland is called the master gland because it regulates the function of other endocrine glands. The term is somewhat misleading, however, because the hypothalamus influences the pituitary gland. The gland has many other hormones that it secretes.

The nurse is aware that the best time of day for the total large corticosteroid dose is between: A. 4:00 AM and 5:00 AM B. 8:00 PM and 9:00 PM C. 4:00 PM and 6:00 PM D. 7:00 AM and 8:00 AM

D. 7:00 AM and 8:00 AM The best time of day for the total large corticosteroid dose is in the early morning, between 7:00 AM and 8:00 AM, when the adrenal gland is most active. Therefore, dosage at this time of day will result in the maximum suppression of the adrenal gland.

Which disorder results from excessive secretion of somatotropin (growth hormone)? A. Dwarfism B. Cretinism C. Adrenogenital syndrome D. Acromegaly

D. Acromegaly The client with acromegaly demonstrates progressive enlargement of peripheral body parts, most commonly the face, head, hands, and feet. Cretinism occurs as a result of congenital hypothyroidism. Dwarfism is caused by insufficient secretion of growth hormone during childhood. Adrenogenital syndrome is the result of abnormal secretion of adrenocortical hormones, especially androgen.

A patient has been diagnosed with thyroidal hypothyroidism. The nurse knows that this diagnosis in consistent with which of the following? A. Failure of the pituitary gland B. Inadequate secretion of TSH C. Disorder of the hypothalamus D. Dysfunction of the thyroid gland itself

D. Dysfunction of the thyroid gland itself Thyroidal hypothyroidism results from thyroid gland dysfunction. The other causes result in central, secondary, or tertiary causes if there is inadequate secretion of TSH.

Cardiac effects of hyperthyroidism include A. bradycardia. B. decreased pulse pressure. C. decreased systolic BP. D. palpitations.

D. palpitations. Cardiac effects may include sinus tachycardia, increased pulse pressure, and palpitations. Systolic BP is elevated.

The nurse cares for a client who underwent a kidney transplant. The nurse understands that rejection of a transplanted kidney within 24 hours after transplant is termed:

hyperacute rejection.


Set pelajaran terkait

Nutritional value in milk [nutrients in milk][

View Set

Chapter 11: Auditing The purchasing cycle

View Set

Health Assessment Prep-U Chapter 9 Pain

View Set

Polygon Interior & Exterior Angles

View Set

MKTG 3832: An Overview of Marketing

View Set

Chapter 13 What's The Significance WHAP

View Set

Combo with "Ethics vocab Words" and Deontological Ethics

View Set

Understanding the Scriptures Chapter 3

View Set

Course 5 Sec 6: Display Advertising

View Set